Vous êtes sur la page 1sur 126

--Fp

ANPILN SE

Fisibl CTI ONI C- L

o G

0 09
ia

< 24

N
H 11,10%-ts . Trwilml

rimtiiimploo.

4%,sk

1%-AL S SiLer

20

i 3'19

_~. ..0.n,re.4...

412 ( umpA . eN 5 li

CHAPI7IRE

LES THORPIES DE HAHN -BAIJACH INTRODUCTIO'N A. LA THOR I E DES FONIC .F 1 OSAS

CONVE_XES CONjUGUES. r t

gr if

- :

,. " .14,1 FropiEltd3 d l'apptdation do durnitd. vid; ,' "z Sot E Un .75,,pace vectoriel narra ( c.v.n.). Pour tout x e E on dfinit 'ariple-stiJn de aatit par F (x) -ifEt" 1) Montrez que l'on a Flx) {f E E' ; cn
et <fp:1> ikr 21

Hen

I1211 et <f,x7.'1,slix51 2 }.

ciue F(x) eJt 11-041 vide, convex2 e: Urn. 2) Montrer que 3i E' es. t str5zel:mune convey_e, atora FW est 'Ir

(On rappelle

e.via. est etrictement Jonw..= sd pour tour oc1,7111 f,g d.1 11 c.: on a Uf + (1-t)gll <1 )Z.1,1E).

ospacr1 Lel que 11 f

3) Viontrer que l'I on a laleinerit F() ilfEE' 4) En dIduire que <r(x)-F(y), x-y> >0 Vx,yE E. -Tlire, plus prcisgment <1-g, x-y> o Yx,ye E, WE F(x), <f,y-x> Vyer,).

LI

Montrer que 1. 1 pn a en (ait <f-g, x-y > ( x11-15111 ) 2 y e E, l'eq E F (X) 1r)

rl

5) Ort svppost a riouveau que tels Cuy

est strictement convexo. Soien x i yEE

<F(x)-F(T).
l'Ioncrer que F{x)

O.

S.bit E un espace vectoriel de dilnension e i) ixi.c de


..Cf

ere une base 1 Gn censidDn pose

E.

Pour icE E on crie

x.e. avec x.. t tt pulir fe

e.>

1) on rpun t E d la norme
TI U

xII1

[21a norme duale f

d'un Illnent E E E' b) Chcereliner explieitement Vensehi.ble F(x) CappliCatiarl e dualit'5 ) pour timit xE E2) Reprenda les quest i'..erns a) et b) 0 a E. est luni de Ia rilotmo

Max. 1 x . 1 . i<i<rt

3) Reprendre lis questions. a) et b) si E est eintn de la nprne

Tizo l= ( t-i
(t plus gnraleinent de la norme
rt

1x ii 2 )

\ili

fixn

lx.I P) UP avec pE

,cut

P i-] 1

Si ot Clf0,13) = C([0, ;

wuni de Itax 1'11(0 . tE[0.]]

no me usuelle

a
On considare E = {LIE cao, 1. 1) ; u(0) =01. de surte que E est un sous-espace ferm de C([0,1)). on consid-are la forme linZaire f : uEE 1.-41 f(u)

1 u(t)elt.

1) Montrer que rEE" et calculer IIf El


2) Peut-on trouver uE E tel. que ilu 1 et (u) 11 f 0 E , ?

-1_1.4_11 I

On considare 1J espace E

(aspace des suites qui tendent vera O,

voir exercices du Chaptra XI). A tout IlIment de c o , u - (ul ,u2 ,u 3 ) on


1 u .

ssoci e

f(u)

0p 1

n.1 2n n 1) Vrifier que f est uae forme iinaire continue sur E et calcuter IflEs
. E. tel que 2) Peut-on trouver un llment ulE

r
y
-1 \lote 1.

Ilu II=1 et f(u)

1.5 1 Soit E un a i v.n. dr dmension iufinie. de E telle que 1) Montrer qu'il existe une base ulerbrique (t.). lel

On roppelle qu'une base algbrique, ou base de Haml, est un sous-ensemble (e i)ie, de E tel que tout xe E s'crive de maniare unique sous la forme :

E E E

x . x.e. avec i12.1 1 1


(Utiliser le leme de Zorn).

JCI,

2) Construir une forme linaire sur E qui n'est pes 'continua.


3) suppose de plus que E est un Banach. Prouver que 1 est non dnom-

brable. (Utiliser te Iemme de Baire, voir lemme II.] du cours).

1151

Soit E un e.v,n. Montrer qu tout hyperplan de E est soit

soit dense dant E. (Si f est une forme linaire sur E qui n eet pos continu
on pourra montrer que f ((xo,r) , ) Yxo E E, Vr > 0) .

A
n

1.7

Soit E un e,v.n. et soit CCE convexe.

L r
L J

1) Montrer que C ct IntC sont convexes. 2) Sclienc x e f et yE Int C ; montrer que . Int C YtE (O. I tx+ (1-1)y.C 3) Et dduire que si int C*95 alors

I.
1

Int C.
-

I.8 1 Soit. E un e.v.n, de norme E E. Soit cc:E un convexe ouvere tel


_que DEC. On dsigne par p la jauga de C (vi ir lemi-ne 1.2 du cours).

1) On sunpose que C est s yaltrique (i.e. -C C) et que C est biNrn. Neintrer que p ett une norme sur E et que p es C. equivalente II O. 2) on ooptid1re tiaintenent E C([0, 1 ]) muni de la
Tit rae

Max 113(01. 0Ei0,1]

Soit
C
de

1 1

{uEE ;

o 12 d t <

Vrifier que C est convexe, muvert, symItyique et que O'C. C est-t -1

Dterminer 1 auge p de C, Mcntrer que p est une norme su'r E ; est-elle quivaliente A II
d

R4hn-Banach em

dimeneion Seit CcE un convexe, non vide, riel

Soit E un e.v.n. de dimension que 09C.

Orx se propase de montler qu'il existe toujo:urt un hyperplan qui slparv C et (O} au sens large:
Noter que i) On ne feit aucune hypothZse supplmentaire sur C.

Tout hyperplan est fernI (pourquoi ?)]. 1) Soit (xdrovi un sous-ensemble ddnoenrrable de C, dense dans C. Pour c`imquit u on pose ennvf xx 1' 1 x

E
4

1 1 1
r
Jm.

(Le . l'enueloppe Ct.:VI...e:le des pOiTILS K i ,X2 , vact et qu ]

). Vrifier que C

st en-

en. e 5.t dense dan5 C.


n E' te1 que et <f ,,x> ii n C . n

Montter qui 11 ex.ist1 f t


0 1. 1

3) En d duire qu'Ii1 existe fEE I teL que


f fl
1

et

<f.x>

O NirxEC.

conclure. 4) soienr A er B deux sous-ensemblem d


cqnvexel, nnn vides et dis-

juints. !loritrer qu'U existe un hyperplan I I qul spate A et > a u 5ens large.

FI-101
sous-enser:51e .:1..)

E un e.v .n. et soit I un. ensemble IV indice9. de et un sous-ensemble (ad iex

CID

se donne

MontreE que les

deux proprltt - s suivantes sant lquivlentes (A) II existe fEE" tel que <f i xi > a a. Vi E 1_ 11 Eiste une con5tantl 14;10 tel La qu pnur tout part it frie (B) et toste fazilie de z- e15 112..7 on a

ti-

GJ "

e.a.im o y O x i l je..1

hontrer que

peut cboisir le E' avec II r II c , N..3ans l*implicatinn

(B) - (A).
l i es-

(Mur vrauver que (.8.) `;` (A), on p.ourra. ccualencer par paee vect.Oria enCridr1 par Les (x j..0.1).

71i
n n

sclierit E un e.v..E... et 1.1> 0. Soient (1i) u<5.11 n ltaents de E' et r_els


Montrer que Les deux proprilts suivintes sin lqui-

valeutes Yu>0 3x E E tel que

(A) fix 1:1 M+e et c a. Vi- 1,2, al

11 ,12 i 1M (3) I i 1

o
1A1

rlels.

a
(FOur prouver que (11) (h) crn pourra Cummencer par supppser que les (.,) sont

linairement indpendants et a l inspirer d 2 la dl=stration du lemme 111.3).


Comparar les exerciceS LEO, .1.11 et le lemme 111.3.

.1ry2-1 1032 .1611.1:101-1- r.1.. 4

(1:rri
Cquivalentes
(A)

Soient r unespace veCtoriel, f l ,f2, - p En n formes linlaires

sur E G.E a l , a t -ER lixs. Montrer qua Ies deux pro.prits suivunte5 ont

11 existe ): E tel que fi(x).-csi . VI- 1,2,-.,n.


Peur toute suite 9.1 ,62,--,Bn de R Lene. .que

n on a aussi

la.. f -e -1 1 L 1-1

1,

A. =G.

i-1 "

7711 1 .

Swit

-R11 ; orF pose {xgRil .>

Soit

hE un sama-espaca vector iel de E tel qua

co)

Mntrer qu'il existe un hyperplan H de E tel que 11= H tt H n P (G.] (On pclurre cormi.ericer par prcuvar que 1.1

(I Ton pe.),.

1,=I4 Soit

P ti

(espace dts suites soy railes voir ezerc ices du Cha-

pitre XI) On Considre les deux eriGetbl .es

x= x= (x )

r>1

Cl x

=0 v-i.> 1/ '

'I I" ( Yn)r>I

E ; Y2E1 rt 51 211-L

VQ> 11 '

1) Vrifer que 1 et Y son: des wus-espaces Eerms et montrer que

X *Y = E.

u
lJ

2) Sot cE E defini par e 2n- I fe 1 2n Vrifier que e .X +Y. 3) On pose Z * X - c. Noter quo yriz=o. Existe-t-il uo hyperplao feroa
=O

`fin> 1.

1 VIO t. zn

L
- r 1._ r
6

de 2 qui separe Y et Z au $en5 large Competer le r'-sutt2t obteau celur du rhorme 1.7 et eelui de 14exerojee 1.9. Zeprendre les munes questions dans E:=1.1:1 , 1 <p <m,, et dans E =c a .

IJ

11.15 1 Salt E un e,v,n. et soit C cE un envcxe tel que DE c. On post * C ** C ; <E,x> t'E VxEC}

; <1,x> < 1 VI E C 1.

*ir _ 1) Montrer que C =C. 2) Qua pcl.Lt-on aire de C lorsque C est un sous-espzce vectoriel de E.

Soient E un e.v.n. et f El avec f O, On eonsidlre l'hyperplan d'equticn. 1f


(t)

On se propase da montrer que pour tout xE E on

dist (x.H) = Int


YEH

r x> IIfli

!) Vrifier que 1<f,i>1 C fifil dist(x,H) v:EE. 2) Soit u E 2 lig en noterit que y

E E
tre I.

<E i x> - <5,u>

H, an..ntrer que

dist (x.1.1) < 3) Prauver (A).

<flu

bit

V2cE E .

4) Uterrainer I et Yetrouver (') l'aide de la formule (17) du Chapi-

IJ

i. 5) On prend mintenant E* {u E C ([0,1 ]} ; u(0) =0} et < f pu> al I u(t)dt. 0 1 Montrer que dist (u,H) * I j u( t)dt I , VuE E. O En dduire que si uEE et u4711, alors Influ-v: niest pas atueint, 414

17771

vIrifier quu les foactions 11R..-*

CW1iniES c-essous

sont ConvexeS S.C.i. et dtermrier lea Eonctions ConjUgu..e (011 pourra auss traer les graphes et hachurer pigraphetz).

V a)

W(x) ax +b

avec a,h11.

b)

t,p(x)=e11.

- d) (P(1)

e)

(x) = 14 )

E)

x)=

/b)

i(x) =

oE

<p

<an

i)

9(()

+ v.. mtrax(x,0).

si

o oir l . x < 0.

1 <F

j)

9.1( x ) =
si

Ip x x

S sL

X > O oil 0<pi< I x < C.

L)

11'1 (X)

NI 1( 011-0+?

o |<p <cci,

my]soit
1) SuiLIL
t *

E un e.v.n. : E 1-de.,] des fonctions telles que (poll:p. Montrer que

2) sot
ce F(t)>O

]-= - ,+92.1

une fonction convexe s.c.i. selle qi- F(0) F )

VtEit.

pose kp ( y.)

Montrer que 19

est convexe s.c.i. et que 411. (f) = f(.If 10*

U.:171

Uta

, (voir exerciees du Chapitre XI). Montrer E. e avec 1<pCce E

que les fonctions (pi. E

1..,+.) dfinies cidessous sont eonvexes

et

dterminer les fonctlone conj u gu615on

note

x = N I ,x 2, - ,xn,--) 44r1
/ OCki2

1111

si

III xic 1

2 <lral

+14

ainon.

b) (vrifie.r

= k= 2
que

lxk i k
tette serie est convergente pt.ur tout
+111. l xkl si 1..i I lx ! <4'21 k x E E)

c)

i.i(:1)

sinon,

EI.20
Qn

Soient E F.

=112 et C {(x1,x21

xl>0 ftt. X2)1'0.

dIfinit sur E la fonetion

- 472
+.
1) Montrer que ti,

XE C si X9 C. est convexe s.c.i. sur E.

2) DIterminer 0 r ; =01 et la fooztion **I

3)

Cr introduit l'ensemble (Union] ;

calculer les quaotits


Inf{Lp(x) +Ex)} et Sup { -q (-E) - 1 (f)1. XEC 4) Comparer la conclusion du fEl

thorlme I.l1 et expliquer la diffIrenee.

lo

SoLutt E un e.v.n. .et A= E kifi ferm& non ville en pose 1,9(x) = e.ot ):,A) = ii. I! 1%-a 11 . alA .0(x)",p(y) 1 1)Vrifiet que 1

x -y a Vx,y E E

2) On suppo.le que A est convexe. Montrer que tp est COUVCX.e. 3) inv ersement prouver que si 11 Ctit COLIVeNe,
4) blontrer que 0 (IA) +

A est comexp,

Et (pour tout A, nrn ncessairement coolvexe).

1.22j Iaf-convolution *
, ], 0,4--) on dfinit Soit E un e.vin. Itant donnles deux fonctions wi p :E -4 1'inp-conr.5ZutUin de ro et 0 de 1a maniare suivante : pour tour xIIE on posa (P v
Ot1 r2narquera que
(x)

rof {1P{x-y) Y)) YIE

i) CPVI.1)(x) peut pretdre (0v ik) (x) <4.u> 51 f


1) Dn

valeurs
ci ( JJ)

LeUle1:1,27: 1- 5j- Ice D(1 )

supp, ise

D((..? )

)*O. Vrifi.er que. (1,7 1:11..) ne

PIZTIC 1 jai

valcar

at, Toontrer que *


0.2 12

tJ
k
%.2

2) On suppose que D(sP) n D(1) . Montrer que


* * k (19-!-1.5)< (q V ) sur El.

ci

3) On supposo que
que (1) solt continue En

IP xa.

et

sont

et qu'U existe xo .e

D(4 5)

) t el

montrer que

0+0)1 ' (IP* 4,* ) sur


4) On sup pose que 0 et
411-52

EP

ti
D(p)n DG) 4 0.

sont convexas sfe.i. et que

Moncrer a

* * * ) = 0:4 0 sur E. (O I
Etant dounie une fonction cp : oo pose

~~

CpittL.P= {151:i l}E E x11. ; k.p(x) <)L}


5)3 ,rvi. (ler que l

s t conveXe. si et se.1-51Pn'eU Si ap151

Ersl

51)11-1 5-en-

sernble COUVeXe de E x11.. 6) soient 4.11,14, :E Montrer quc epit(1.15171:0) u (epist 1p) + (epist .1) (nette 51 e end.
.311

-cn,-HEJ des forte tions [elles que D(1.15* ) n D(1,1)

sens de Vaddition alObrique

sous-criswaibles

7) En dduire que si (0.1,1?

spnt des fone lierss corve:ces tintes eXe

que El (ip ) n (14, )1 1 .S5 F alors (1p911,1)) est une ro riction

I.23 1 ,F,Vgz.11.4.2-2:5ation par inf-cor.vauttion.


Soit E un e.v.n. el eQiil 1.P: te l qbe que : (I) rour tout :E
1

1-i41 1.01] une lunctiml comete

-.

.repose dE etinatruire une suite de .f onz. t las,- ( IT) Cell e

, 04 es t corvexe e t .tonrAnUe, eroissnril vera 11(x)

pour tout. x t, 4.1] (x) ciOnv A c. t poSe

1 ) Montrer que E -4, I

(X) = Inf x-y +4,5.(y)]. TEt peut trouver DI as sea Brand te l que si n> N' RIOTS

Dans t4.ute la suite en prendra n>-9, est zonveyle (JOit lutxercice 1.22) t que

Montrer que 10 11

11Pn (x)-1v.n ( i ) I n I Ki-x2 3) 1I.5.terairpel' Wiri) v.ltrjer


que 1.1:In (%) <1p(X)

Vic E E l

11 .Dntrer que pour te.ur X E E la

suite (IP (x)) est crissaote. R choisit ynE E rel quC S) Soit n(9) ; 1pu (x)'Cnilx-y -114)('51 ) r- (x) u.

; - c) Montrer que lim y x et en di"J.duive que lm(x) iii)( 11-F, Tire, n (x) = -2-u ton pourra ra.isnnner par l'en- que lti 6) Pe-ur xg b(Q), izslintrei. n wat).

IA1L] Sem7Frodui t acata ira Soit L un e.v.n. 1) E-0 t 4.0 : E I`


-45 1

conmexe Etnt donns x,y E on considIve la

funetion
h(t) 4 1(x+tv , ) -L(x) t

t >0.

VIrfiet que h est croissante $ur )0,+m( et en dduire que 1im h(t) Inf h(t) existe fans [c,4.54. t40 C>0 on dUinit la scmi-prcdwit zwc:cire [x,y] par

[ [x,y] =1.nr - >0 "


2) Montrer [1 .-"E 3) 1.1;.Intrer que [ n `1; II XII y

tyll

-1!

n2

1.

CX
et

1xh 2 + Idxs yl

.c,yG E. Nrk En, Vz

1. o

[kxPy];',Idx,y]

+x, 'E F, V?O, [x,y) est convexe.

4) liontrer que your tour xE E, la fan tos y lienatrer que la fonctitin G(x,y)
5) ,

- ['x,y) e$1; sge.i. sur E x E .

Mbutrer que Max <E.y> seox.y EEE EF(x)

o F dsisne l'applcation de dulitl (voir eprollaire ciee et

du eours et exer-

?curra posar a 31 lx,y1 et aNYliquer le thorame 1.11 aux foncLions drinies par ttl(2) = 111:1:+z 11 2 x 21 zIE

r,

et
- ta
pouT Z

ty ce t> O

1(z)
+4

si lleurs.

1 1

r ave.e 1

0 lAteumnec [K p yrj lorsque p<':ED ou bien fi x Il m u

ffillitil Cst rruni

da la n'orne

DK.

J 11 IN 1 P\11? d. 'ji i

Mala ix. p.
lin

(on Pautra s'aider des TSUltats d 1' exe.rcce 1.21.

> 251 Norrnoe

e t fariet.ilon5 atrietentent colipezes.


eSt strieleMInt COTIVE%e( 1 ) Si
I 11

Soit E un e i v,n, OrL dit que la norme II II

tx. + (i-t)yli <L o 595:s yEE

ario

y1 1Ix 11

y11-1, Vtl]O r li.

Ori
r

diL qU'une feineUrph q :E

est strictement COETY5X 5i


YX, y E E ave} x v-

11(tw,+ (1-1)y) <t1.1)(x) + (1-1)1,Ky)

y,

l'et E )0, [

1) Mtintrer que 1a norme 11 11 esr strictement conveKu si et seulemeot Tri la fonetlopi ) II X a 2 s I strictement CetrVeSe.

2) 1 1 .1me question avec .1,9(x) - 11x 11 1 et 1 <p <go.

r711.3
r fenr:5, Si o

Soient E et F deux espaces

Banach tt Selit G' E 1:11 S.Ciu5 - 511=07-E

T :C

F une arplication UnInilre ContinliC.

}::pose d r..,Dritrer u1 L 1 ni emi.ste pa wujenurs T :1 7.

lindaire zontinu. A cet

effet on

eppsitlii-re nn espece

Banuch E ec

sou,...-esp.2ce lerm1 OcE sans supp1menteire topuIlnique (vlir la remarqlse du Chapilre 11), Oy. ,:rand F G et T

I. MQnever que T n'aduitt aucun pvolonztoiInt

T :E 4. Y linalre contnu. (On pourra raisonn2r par 1 r absurde). Comparer a la cone1usion e turullaire 1.2.

IIII.1 I.E.i

(1 ) D4n5 ce cd3 Sri dit

que E est striclement carmewe.

CilikPITE I

L
1) Noter que l'Istaitl 11 x11 2 entraIne II ?II] II 1 p . F(x) est non vide d'a,pr:13 l corellaiTe 1.3 riu .ccurp. ti et ciair 1 gtine la Esconde fcitoe de Fix), que F(X) est eninvext et 2) Dan un
E i v i rt.

strieteuent conveze ur ensemble couvexe, nen vide tt


.5E1,31

centeno cians une spUre esL rduit un 3) Eoter qi_te < fi y> < f Inversement, vippcsons que (1) y II
2 .2

point i

J
2 .

U y 11 <71If fi +721,11

leve E.

(1) on elLeisit d'aberd y <E,,r.> lx:1 2

2C avec Ielk ; en faisanL varier 1 t P. viera >O 11 vitra

On elniejle encuite dtfis (I), y av.2c 11 y P <f , y

<16 2 .1-11 x1 2
2

Par consIquefit 451I f II - Sup <E y> yE II y1=6 On connIut en prenant 4) Si fE.E'erz) on
X i[

Al
et si gEF(y) on a

Ir x11 2 - 1-ylE 2 < < gpx-y 2 2 Par adclitic.in iI vient <f-g,x-y> > O. Par &j'Acure on notere que < f-g,x:y> = II x11 2 + Il y1I 2 - < f,y> - <g,x> >11x0 2 +11y11 2 -21xE11yg. 5) D'aprls la question 4) on sait dja que lix.11 = bit . D'autre ps-rt en a <1"(x)-F(Y)1x-y>. [0)(11 2 <r(X),y> et les termes entre crochets
5Drit

C 11)(11 2 <F(y.),x>)

>0

Don:: 11 x a 2 Ni 11 yll 2 1.<F(x),Y> fi P ari v(x) g.F(y) &dee

< F(Y) ,1>

ce qui implique F(x) E F(y)

la question 2).

afll

Max 1f 1 ' 1<inn j 4n. or, a si x*O '

f E r(x) si et seulement your cha;ue

f =
i

rbieraire dans l'inzervalle [-1>:111, 411xlii) si. x-=0.. 1.. 2a) 2b) i=1 Etant clonn xE E cm considlre I= 11 <in ; ixi i = 11x11 .1. Mora f E F(x) si et seulement si on a U) fr, 1. = 0 (i1) f ix. i >0 3) Vi g 1. Vi)- 1 et l [f u i . 6x11 .

iet
I f El .( 1, 1 le.12\1/2 E' is,i.L ,/ I il

et feF(x) si et seulement-si oil E, =x, Vi = 1,2, .... ,n. Plus gnralesicnt on a 1 1

et f E r(x) si et seuleraent si en a f .

1 1 I f 0 ET = ( i If.IPYIPi avec -+ -1 P PI i=1 1 i . ipl-2 x1. . '1 , . 1,2, Iix IP -2


P

*n

ErE
1) 1 f 11 r
h1

e notar ve f(t ) 1.1

1
1493 ' 1

a >0 .

2) S'.l existait un tal ul E on aurailit

C1-u)dt =O et done u E 1 - absurde.

1) Soit 1' la famille des souG -ensenbles de E ln'airement indpendanta.


11 est lacile de vebir Toe P ordonnl pr 1'inclu5Un est indued-t. Gr:Ice at lezne de Znen. P adoet un - 1.5.mant maxlmal ; at est une base lbrique (ei) tei. Count e *O Vi E 1, (pul peut les nongaliser, et sumnser

... = 1 Vi- E I.

2) Corone E est d,2 dfinension "J'Unja. orL paut suppnserIwL n exiata une faene 11 aire f (et une sevl e) lene que f(e i )-i si iE11 fiCe.) =0
3) Supposons que 1 est dlonnbrabIe, i.e. 1 vectorE1 enendr pp.r le$ (e.) el: > Alorb On consid1.1.e Fri =etpaee ezfrreien XI est d'intEr.leur

est feria d:

U
n=1 11

E. GrZw-e E

lelizna da 3aire it existe nD tal que r rLD -5bsiJrde.

D' al E. P

Sbient x, yC C. alorl
O (1'

et y = limyn avec xnk yp E C. bono VIE , 1 O,, ti. Soient

(1 - t)y= Lica [tx

(1-0y] et par Suite tx+ 0-ithrE

x,yE Int e ; aloes i1 txisce 1-.). O te1 que E(x,r).=C e t 12.(y r T)C. Par suite tE(K,r) + (1-1)11(y,, r) cC Ye ~~ t13.(5,r) + 0-013.(y,r) =E1(tiz + (]-t)y,r) 2) Soft r >0 tel. que 5(y,r) C. On a + (1-e)Idy,r)c17., Vte [0,1], e " est-a-d ira 15(tx + -t)y, (1 -t.)rj CC- Par consquent + -t.)y E InLe Vt [O, !1 ,..
n n

[0,1],

a E

- 3r.31 . Or 3) On fixe yo e C. Eunt dogal x E C on a x =(1 -4x + 1

1 )514n:1e Int C eE (1- 7

done

Irle C.

On a cinsi prouv que CCTM 111 19 r y

; dora
"Cc Int C.

Ie
1) On s dt dj que

t. r

p(Xx) = A.p(x) \ni> O, I1 reste prouVer que :

VxE E et p(x+y)

p(A)+p(y) slifx,yE E.

i) p(-x) = p(x) Vise E ; ceci rsulte de la aynitrie de C,

(p(x) o)

(x =C1) ; ceci rsulte du fait que C est bort ...6. Plus pris.cislL Vxe C. II. est facille di voir qua p vxE E,

mont soit 1~> O t.l que nxli u

Pot
II u

r
2) C n eot par borr ; par em.mple la suite u (t) i varifie u IEC st n n Uhnt
1

On

p (u) U.

lu(t) P dt)I

./2 1

p est une norme qui n'est pas Zquiva-

r
L L. L.

lente

la rzme u

M IL 1

1) Soit (X io .k r ,)k n)E11.71 ; Vi et i-t


de gene que P et un compoet de En et C n

=
J

est i t ivage de P par i'opplication.

2) On applique Salla-Baruch, 4.1ela i..upe forme zoritltrque

C et {O}. Chi

-pare en et 103. normalise cnsuite la tormg 1iu aire associae l'hypzrpIan qui s& 3) Evident. 4) On appliquft ce qui prcnie C A - B.

I . 10
(A)
(B.) (B) est trivial.

(A). Soit G P.:space vectoriel ensendz par /es (c.) iE1, 2tant dorin.1 8,x. et on pose 100 41/ Ca EJ X 1i jEJ "

xE G on crit s.

On vrifie, grlice

1.1hy r,c,ti-ise (B)

d'HinitIna a un sans et

'que I g.rx) 1 JQ II U a VxE G. la prolonga ensLta 8,1El l ade d'u Coron aire

n 7 '. 71 (A) -1 05) Est trivial..

(10...1.M.cluniplic,se~clueles(f.)sont
On pose el v. I. , cc s u ) 1E.71' at on conaid.lta

Iinvairement ineMpendncs.
d` -nile

P applicatirm p : E N-

Pr 1 15'lx)`(' C flpx>i<f2FX > 3 5.DiE C=11.E1',

.<1.1. .x > )

1151: hihc . !. Crt dlercha 1nontrer fue E 1P(C). OE IZ i-5ornrk On epZire (P(C) et (z) sens 1 .511-81

par l'absul:da et ori 1.1.ppoSe que al 4p(C) (voir E:cariz:Ice existe done

o tel eiva
Ilia l x5 C. Par suite _J =O. Cauna 1.42s (f i ) c qui t a e Al2. 51Idt ind6pen-

0.141(01:75. 11 . v, C, i.e. < Z I e iliix> 01+s) Ii

ii

II <

L t

a 11 hypeithaa (3)

sunt lnh.:terranr. inLit . p.endents Dans le e;3.

r'n "2:4.trait deld (y un systL:na

danr rr.lx5indi et on lui .apr.liqk:le ce. qui prts.c11 1 -2,

i) IL Est clair que C.=

skg tt que C

est ferm. +1 9kke. s&pare INO

4* IntriSIortt Supp4SonS qU I il existe xa E C et C a szins etzict ; il eX7,13te darte .E gi,

et %GER tei que

<f c,x> <Q,D <


Comm OEC, a turs

> G ; pasarit

Vient

<g p x> < < <f,xo> VxEC.


DeJnic g C et ci, .boatit une contrarliati.Dn car 9:1) C 2) Si C est ion sous-espace vectcrial alars. C ={. .E' ;<f,x>= Q V:tEel.r01.

9.P

1) Voltee quu si yEln, alQrs I <f,x> 1 -| <f,x.7> 1 2) Trivial. 3) 1-^Isuite de 2) que Uf 11. Sup ueE i1.f11x-y 11 .

kf u>
II piE

c . r %), 1 dst(w,a)
Si

4) (deux

(I )41. (g) 51 gOlIf


512nt

KPrintS

linhires qui arte le EnZrne ninau crlt aloes

90 ir

L formule (17) du Chapitre t

dist(x,I-1) nix <g,E> . gEllf 11 5) Ori 3.wit (voir olerice 1.3) lile li 21 Slinipo2;rib qu i 11 in.iste ei; Une distru,11)= 1 < > 1.

ven tel que


ih.L-71! ifu(dti

disit(u,1I) A.loTs un a (1, ,rgee i(*)j. 2tt2 1.1 .1.1t1

Dote

:fax lult-17(z)[mi.r[u(1)-V(t)Uti | carlte u(01 DV(e) tE[Q, 1] n'elt pos 5 ible que si u-vE0 e i_ dcFne

-b a) 40 (f) .. -,-f b) (e)


r)g,

si e =a si f E f *a,.

si !>0 si f C1 si t <O.

e)

(f) =

[e!. 0.

i)
e).

4 1 (t) -

Si C. q> (1.1= . 1 4cp - -1011,1f1 si |<O.

(| i 2 ): /2 . 1
11)

1 O

pu

si

si f <O ,

4---111r1 Pr

gi g>cf
II E .

+11 Cr'.
11 + 1 = L par : P 1

r.19 I Le5 Unctilm5 conjugwles sont dfini9s sur f a)


T k=|

1z

+p. b) 14' (f) _ s. a. ik,((k -1) .g I k= 2 4-12


(k-l)
ski al( f

(k -1 ) <

k-2 sien,

si c) -)

EME 1) Evide.it , 2) - I g A
1

[L,I2] ;

et 4fif2>

3) Or. a
1111(0<x5 + (x)}
eE

s,* .i

tir ,`

1 tIfi,f .21 ;

0}?

( -Z) 4

1.

E E 4

et par suiie

Sup {-1,9 (-t) tEE' e

(f)} =

'XI I
n'exilte

Les hypotIlLs25 du 11-t&itine I.I1 De seint pas satisfaites : pes d


poin't

e E tel. que 4(:)<1.-, p(Ec) <+-

et EP continua en xo.

On a

E x-a II < II x-y II + 115-3


preriant les In! vient $[e} v.- y II + k.P(y) iELEA ILP(I) ~My) x -y 2) s'olent. le.,y E et t E I0,1] Etbangeant x,y on obtient

Etent do nn

> O iI existe e EA et

beA tals que


U
LI2nc

-a ]

<1,9(r) +

E C

y-b .c.p.(y) + .

(1.-0 y - t La

(|-t )13] II .1 up(x) + (1 -'013(y)

e.

Cr [ta + (1-1)ni E A et don


5I(tx (I -t)

ti,P(r) + (i -t)q1(y) + a A {kE E | 1.1)(x)

VE>0
et done A est tanvexe

3) Carrime A est ferm

DD

si

est convexa. A) On a

(E) = Slap{ < f


geE

- Inf II yt-a II 1 E.
agA

2x]p sup{f,x). - 11x -all!


).. .EE AA Sup Sup{ < .3; > al :4 E (f). 1.)* (1.) + IE
Er

1. 22: 1 1) SoiL ferp(.p#1) n D(17) On a Vx r y E E < x-y > - 11)(3.:-V) <f,y> et par addition vient 0-9 IIP) (X) < f > IP (f) En. particulier (951 1) ne pren4 ja-nais la valeur ( (f)
On

- 41 (y) <0 11 (f)

ci

Sup{ <E,x> Infrw(x-y) +1.11(y)] } yEE wEE Sup Sup{ 4,9(x-y) - (y)} gEE yEE Sup Sup{ < f,x> IP.(x-y) - (y) } )E rp (f) 1.11* (f)

2) Remarquer que (04-2) 11 a'allt da. ylr.ifier c sue <f > -449(x ) Yxe F on Lj, (1: ) fj

Ceci est 1.1.t:ide.r.r s l'on crit f-1,7x>+<gpx>. 3) Etnne donn (1) Or Sup{ < f 1 z > - u(x) - 1.5 (x) 1 r- - Ird {";' ,2 (x) + qi (i.:31 aEE faut nontrer que

Stip{ <fpx>. - LP(x) -I(x)} . Inf {0 11 (f -g) + 11:1. (E) } , ZIE OE'l

1
xEy.

avec 441(x) .1.0(x) - < f 0: > . App1iquant le thlirlmt 1..11 aux fonctlors j] et -4:.. on obtiene
,-,.

Inf to(x) + 01,x)) a Sup su{ 1.-(p (-a) -.0 (g)1, itlE ce qUi currespand prcialment (1). n
9 en a

r ...-It

1 14

4.911 )11 (x) - sup

< 1 x)gE

Ltp

(Eg)

(g) 31

rsEi

P. Su p. Sup

<E,x-dl(f-g) - lir<s))

Sup 1,11"

x>
* go** (x)

-g) - (g)1

5), Et) c.t 7) sont des questions faenes.

L.
772-51
L.
14(y

on sajt {Noir prelpositio.n. 1.9) qu' il Existe LEE' et OElk tela que Vy E puend n> if , on a qle.(x)?-nIIKII-C).-w..

2) La Ewnetion gpri est Virlf-crinvolution de deux Eorsetinns convexes ; elle

est done convexe

q uEstion 7) de l'exetcice 5,pi n (x1 )-rinti(x2 ) 1 n 11 x 1-x2 d


DU

raisonne t inmune 1.11 quCS-

tiani ! 3)

ltexercice 1.2!, rt. ) n$ E , +49 srlt la luestiz,n 1 1 e llexercice 1.22..

5) IV E:J.1,113 la .qua tior 1) pti. a 1451,y) n II x-yn II < Ork

-1E1 II y II - C, Vy E. Di 5:11

vient

!I II yn II +C tP{x.) +
'Sc

d.Lluit d'abtirCI que IIy n ll restP. bDrfil guara t. (Ji dipu tp(x) L.D(y

et pas que ji

111011xy

-- et con-n.9 la forbIti0E1 1.11 es1 S .e

1r~

ca. tiro ip,(x).(12(11). trk. 6 ) Supposgns par l'_ J surde qu'O. exisn arta constarite C Lene q' V. On choiint y co=pee la questiun 5) EL GEL volt qua1.. y et d.j,ne wr.x ) 1,pryn)<C ; ce qui St absurde. On

U,.

Pcur ctlaq ue t >0 f ix la f.brietIon u


_

y 1-1, est eDuvexe. Done La foneticin y convexes. DT d tre de fono .L

xi-ty i! 2 -

o 2]

tx,,y1 est conexe cc .e Ifirlit.e de fonctio.ns x+Itya -1: 2] est cae sup

G(x,y) = $up - 27 t>0 cumtinues.

~~

5) en sait dGj. (voir queselon 3) de 1 1 txeretce I.1) que 1 2 > <E,ty> Ix+ty11 2 -2 lix11 et donc bc,y1> <for> vx,y F., YfEF(x). D'zkutre part on a (f) et 1 o si <f,y> +n40 #k (f) * <f,y> +EL>i). I 2 -<fix> I 2

de voir que Infhp(z) +1(z)1 -O. On dault alors du thtt- Ime WEE di re <fo,y> DE a et e l est 1.1 existe fo E E P tel qu qii (f0) + 4 11 (-f o ) . 11 est
f acije

<fo,x>

I1 en rsulte que Ufo!' fix II et <f oi x>

xE 2

'
1

lJ du.ne f e F(x). 6) a)

1 <p

,y)

11x11P-251y.
Ilic II P-2 P
(Sign

t) 2211, tx,y1 12 11 }: 111 { 1 i c) p

xi )y i +

1 ! , 11 ,v .. 1 1
1.

[K,yr]-Max(x.,y. ) ot.. ' I- {1 i.

irL ; [nil 11

bIL1 Soi:t T

, T. O vrifie F .in prolongtmrtt linair tontinu dc. (Y) n G{{:11. Dono N(T) scirait un suPpl.laentaire

ais,%erit gin. E N(T) + G et copologique de e - alisurde.

CRAPITPE i/

LES THtORMES DE BANACH STEINHAUS ET DU GRAPHE FURME, RELATIONS D'6THOGONALAT: OPRATEURS NON-DORNS, NOT ION D'ADJOINTI CARACTRISATION DES OPRATEURS SURJECTU7.- S,

contiruit del .ronctl:0?:t Soit "..7 un espElce de Banach et Boit 0 :E - ] 3.c.i. Soit xc e Int D(4. 1)Montrcr qu'il 6145te deux const.tnrts R>01 Lenes que
] 1.11bc fOrLCtCl. Celrxexe

r
L..

kpi(x)

VxcE E

avec !x-x.0 U R.

pourra introduire las ensembles

p et 11(x)nli.

2) Iktcyntre7 qui?. Yr<1, L 0 tel quo

.00:1)-(,9(x2)

x1-x2 , V.19 ,x2 E L r

aves II

-DED

r,

.2.

Plus prcisment, on pei.t choisir

Soit E un espace vector'...1 et soit p:E 11 une fonction

que

i) P(x+Y) Z p(x) P(Y) Vx,YEEk


pcur xE S fix la fonction

peue) eSt cQrtt irme e R da.r.e 0 pel.z. tout

ni) si una suite (xn) de E vrifie p(xn) --N O, alors p().xn) r >LER. - rifie p-(xn) Montrer que si une surte lx ) de t v1

O tt si int sLi:te

(d) de Y1 vrfie ci

alors

Li

ri

p(el

) --h

O.

t ant r.nnl 5.50

wri.

pourr:. Ittre.duirz les ennerbles F a = {1.ER

IrDi

Soianc E et

deiux espaces da lanach et soiit (Tn) une suite de verv un

:(E 1 F), Or supposie que peur duque xE E, T x converge quand n wirelfinite


r>1 x dans

u
0

IlipntreT ve si >in

E, alors

Tx dais

F.

u
EIT7Y1
Soient E

et F

espacts

de Barsaz.h. Sede a(51,y) :E x F 5, 11 une

forte bilirlare cene que


i) pour tout

xE E fix , l'application y q--I-

est tontinue, e(chy) es clAtinue.

ii)

pelir rout -/C-F fiY, 1'epp1inatIon x

Y.entra7 qu'a indste une canstarAt nkell talle quv


vyCr.
f In pwi rr, irty-1.-Llirl un uOrztawr

.1

T :E --... Fl et proLver

T rlut

opftracur

r4 sr'ke

[n..] St,it E un
tif3 te11e que 'Dm(
n

aspace

de ranach tt soit ( ) une suite da wi2eis

O.

SoL} (f ) un suite de E' v.lril- a17,t la propritl n

{ 3r >0. v. e. E avec :: wIl < r i <f

C(

) el t .e 1
n.

Q ue

x> < e 11f II + Cex.) pour n

a.
1

Mzeitrer que le suite CE ) est tiorna, n fn tOn ligurra introdulre n = + e 1: t n n

y77E1 Oplratow$ ponotonw locaIendlnl


Sien aPP1 'kat ic.n.
E un espace de Batach

brnplo.

et p(A) un SoUl.eft.5.1mbIft de E. On die qu'ffire

D9.) c E I E' est mor2tuno si elle.v&rrie

.<A31.-A-jf,x- y> >0 ]) Soic x


' tepes que

Yx,..yED(A),

Int ti (A) . mr, ,utTer qu' i) elds.te deux constanICS 11.0 et e

a Ax IL <C Vx1, 1.)(A) aveQ. O x1.0

r <R

[0:: pourra rais.9nner par l'ENsuyde et tonstruire una suite Ixn) de CICA)
t.1Ie qu2 n x o et II

1 1 tel que B(xw)<:D(A.). UtiliII --m T. Choisir i> :

ser la mcnotcnie de A aux points x et x04x avIc Hx2<r et appliquer l'exercite 11.5]. 2) Gne'ratiser 1.(1 asultzt de la question pradente
3) Gri.c.3.1.i.ser

xpe Intreonv D(A)].


P C5t-l-

le r.sultat

de 13 guetion 1) eu C35 Mit tiVDCIlie. Un SoUS-CM.Emble =I71 Vide

dire qU

par

E DIA)

AX.

est

e E' ; la rew-

tione s r e<primie aloes par


< -gi.x. y> SerX.Y11:3 (A)
r

iEAx, Y

Ay..

nat 0

a ) UDC luite de rtslls ct swit ] 114 Z-, ifn, suppulle que


1 I lx I <`" R

) de 2.P ,, di..1Dr5 14Dritrer ql.2 a E 7...P

II:Ft...1T 11 d6rinitien de P.P. vcir emareices du chapitIe KT)

soit t un espace de Banach et soit T :E y E' un oprateur linIaire

te;_ que
<TX,K> >0

Vx e Fi.

Moncrer que Test centinu

[On l'tablira ce rsultac par deux pi&thed$


(i) Utiliser l'em'rcice II.6. (U) Appliquer le thorl'ime du grpbe ferpfl,

M:71 Scit E un elp.r.ce de 5..11-12c1 -5 et s.c.t 7-E -4- E' un ol.D'rateur

aire tel que

se.
<Tx,y> .<Typx> 1'lontrer que T est continu.
VxpyreE..

1:4..10

Soient E et F deux "paces de Banach et soit l'E.C(E,F) . en suppoE

que T est surjectif. 1) Eoit !4 un 54115-eftsemb1 e de E. Mntrer que T(15) est fetm dzns 7 s et

seu.lur,ent

5i t.'1

1N(r)

eS

fenn4 dan

E.

2) En dduire que si M est un sous-espare vecturiel dice, yen <co, alors T(h) est fihrti.

de E et si

UL LI

sa ient E un "pare dft Sara' h ' v.,L1 et.

CML) On suppg5e que

T est surjectlf, Mbatrey qu'U. e i te SEL(F,E) tel que T 4, S g. inyer .J.ale droLte. 02 ?es
i-f r11:qucr 12 11-15,cre 11.:0, 1Sis chPrchAr h
explinitement S en

autrtment dit T est

la Voae calzm:qua da L1 ].

II 1

F.

Ut T17.(E,F). On zuppose que R(T) est fern ot que dim

Ort

-le fi : s i.e. considlre sur E une autre norme 1 !, plus falible que la non ixi<1411x3 E WEEE. montreT qu'il exista une constaue C telle que Ex1 4C(lindi *15t1) E F
len poLirra raisonnez par l'absutdel,

Vxe E.

EAL So.ient E et

r deux espaces de Banaeh. Montrer que l'ensemble

ri {T.E.t(E;F) ; T est invcruible a atoche}


est cuvert dan :(E,r). [On pourra commenc.er par prouver que l'ensmble

{Tex(E,F)
est ouvert danl X(E,n1.

T est bijectif}

espCeS de Banneh. !'l " I Solcut E et F deux

1) Sol_L TECEM. une constante C telte que

que R(T) est ferm.1 si el sOutiftlent S l il existe

nj

dst(xj.:91) L C II T II Nix e E. [CID pourra uLiii.s r l'esp3cP. quoti:ent E!N(T) ; vuir cxerwicc XI.,

Z)

BU) cE

' -' 1 "

7 un 45p .re.teLice uluri.-borne et

Montrer que RO..)

r
L

.1s.t ferrel s t et seuli=ment 9 1 ii ixl.s.re une constante C telle que


di.S.r. (11.1'2(A)] C. II MI 1. 1
VLI

CA) .
la noma

E
[On. pouTra. in.tz-du_re l'opIrateur 1' E0

F avec E 0 - (A) man 1 de

du

Gr.pp!-.e. T:.A et appli.locr la question 151.

501.1.nu El, F. et V tr.pis clpleS de Banach. .S0i ene Ti F..e (1:1 j)

r
ri

Ta .q.C(E7,r). On surpose que 1(.117 ; n F.1. 2 ) 1,0)

et

k(71) bRn5 - F.

llontret gut: Rcr1

et Rer2) scrit fe-m/9. dgfirde p9r.

[:11 est util,2 d'Intru.:_ruire l'applization T ztm[:2 Illc1i11 211 .1r 1:1 + TZxi

r_

fr:76-1 Suit E un espace . de


oL15 de E, On supposo qu i n exitice

Banoch. Soient
elDn5t.s.nte

et L deux soils-espoz.fls 1.er-

e t

dist (x...G n Lj c
tiontrer que 1 +1. eAt

(K,L) vlIE

L
- .11 1 de aorta que On (vnir ex.rnines de V. Civapitre 115.

1,1= co co
1.).1teritiner

un sctis-espaTe feri

(xer
et

<E,x -0 vfc..21 ;
<f,x>= o 1fx1111 .

Efe {f

virilier que lq *12.

Soit E C(10,1,1j) mun de sa norme usual Le. On considare l'oprat.mir A :D(A)=E E dIfini pzr

Da) =C1 ((0,1 1)


1) Verifier que T:14A) . E. 2) A est-L1 barr ? 3) A est-il ferwl ?

et Au UY

4) OrL considlre l'oprateur B D(E) E --> DeB)=Q:2 ([0,1]) et


B est-il ferme ?

E dfini par

11.191 soient 2 e,: F

espacal de Banca

dt

soit A :D(A)=E

F un

oOrateur non-born1 aves J(A) E. 1) Zontrer que


et * c 11(k

2) en cuppose

c:e plus qua A est ferarl ; Toontrer que


1 5,11

CA) I. R.(1;

11. I )

Or. aborden, ilirec6erient ces quttions z.43.ns chtrcher


Ir 1 R(A ) tal qul tu,03

reproJuire la cilmerstre-

tion da eornliaire 11.17. Baur la question 2) on pourra taisoniver par l'absure, considlter G (ft) et appliquer 11.2hil.-13anach.

Soierit E un e.gpec.E de Barsach et D(A) .; I,- E' un operatPur


LO) = R.

L'orn aves
1)

011 s

uppo.e qu i n <zxiste une conste.nte 0 telle fue

(P) liontrer que l'.7.(A)c',Z(A ).

Als!u >

C Al:

El 2 Vu E D(A).

2) Inversement on suppose que ti(A)wN(A ). Montrer que si A es t. form et


si R(A) est fern6, eters fi existe une constante C cene que l'on ait (V).

Soient E et F deux espetes de Banach, Soft TEEi n ee soit A t D(A) c: E F un oprateur non-born, fer=6, avec RTIT= E.

OR oonsidre 1 4 oplreteur B El(B)c=E

F dfni Par

D(B)=1)(A), B *A+T. 1)Nontrr que 11 est fenn. 2) Nontrer que D(.G ) DD(A ) et A+

M.2] Sait E un aspare de Banach de dimensin infnie4 On fixe un lment a E E. av5 0, ainsi. qu'ene forme linlaire f : E -4-1ft non continue (de telles
formes existent, voir exertice I.5). On considlre l'oprateur A :E

E dIfini

par kx.x-f(x)a, xEt. 1) DIterminer E(A) et R(A). 2) VA) est-il fer1.4.1 ? 3) DItetmincr A (prIciser avec 20in UA )) * 4) Durminar 1.1,(A ) et R(A ). 5) Cp! ..par,:x N(A) et

1 * 1. ) a) ns i qu.~ no, ) et R (A) .

6) Cordr.pnua les rIsultat3 cbtenus avec ceux de l'exercice 11.19.

ni= On se propase ici de donner un exemple ,l'opra:eur A : D(A) c: E


L )*El . nr, ,n-born, ferrn; avec D(A) -E tel que FUISoit E L1 , da sorte que E' dIfini par On considare I t oprateur A !D(A)CE E

Da) = {u m (un) e xi ; (n un) E 01


et

Au

(n u . n'

1)Vrifitr que D(A) F E et que A est term, F) 2)DIterminer P(Ak), At e: D(,

;
nj

Soit t un. es paca 112 15a.n.th t suit T'GZ(E), G ra.ppelle - voir cotollaire 11.17 - que

Nen
on s

DR(1 . ).

propase de dc.rmer un exemple OCP cene inclu.sion est s.:rete. soit E -.1.1 de soy e que E' &k. 01.1 considl-re VolArateur l'E.C(E)

per n rs>I D!cerminer KT), N(T) , r Der4 ) e t

3
ILU S ephateur T3 1:1 () So:ient E, F et C tris espaces de 111,2nach, 'Sait D(A)

Rvec D(A) = E. Scit Te+C(F,G). .01 -1 considiLre l'apr:rateur e dbfir.1 par T3(73) I D(A) er B.-T a A.

o D.r.ters.dx...,r
.p.le

13 piest pas n'enssaire-neilt

011 /99 5 i A ptzt

VE.16 I) Sien

Soi.:ne E, F et G r i espaces. de lbmar.b.


eL

2E.e(f i G), i.o.ont.ner . : Je. (5

it mr T est bijlttif

2) Cn 512.1.411133.12 que T'EX (E, P) elt bijnt f Iluntre


et que (1 11 )-1

11.27
14'

Saient Ectr deux espa.cns d Benac.li at SZeit IZJC(E,11. Sd.t


"HE'

une

.E Q

ti 1311 e4:1 r-Ve XE

seppose qu'u]. .e.r..1.zte ue, poirit dker)

oil

est .rinie nt apritmre. en pose W(x) = (Tx) x E E.

Eontrer que pour taut E F' *u a * * EP CT f 5 geN(T* , .* (f-). (f -g) = piin gEtl(T4 )

15-

C1-1.APITPE I I

r-.

(m
1) Boit

Quitte { E

faite une transiation en pcnt toujours su:31. - eser que xp =G> II x II T' ; cin choisit P >0 assez petit pcur gua

rKcD,p). Les en5ecnblea F

c
E
r

sone ferms er U F lrr aprr., le lt un de pira rt n il existe tio tel que Int(F ). wi CE PI dp i >0 veis que .111{:c ur p i) wr n -f,,x;+2x) soit ineiutenant x E E avec Ilx1 <-111 ; cit.et 2 2 2 I 1 done 004 11 7 110 + 714)(- xL). 2) Il
ti r

.1,'E E et

[a i li tris que P. f

eC

txi (I-01. On a

.14>N2

1:LP (

t)1`1

et par svite. ,,p(x2) -.;r(x1)4(1-L)Dv-r.p(y.51.1i or et par 1.45(x2) -zig(iEi) <j2 1 D'autre pan si 1. 'on prerd x.2 -O ri vient1.:1 II 11-ts x, II 1 +R --- 7' 21M et II. ( -1 - ) (7.-1) 7 d't5;:1

obtient atora <J(0) -115(x 1) <4.E11.krir:x113 et par siii ce

[ira Nottr
ip(0.0-ht)xj1

que les F sont fetuDI:s et que ti F

Crtc

Zaire, i1 existe no tel. que Int Vti 71. 45 Vorie 1_1 existe ko CE et >e te1 1,5 qul Vk>ria i lit avec iti <1.

ti

Wautre part on note que

P (a kxy )
p

P [('0 "k-a ) xid

i p((o-).0)x.kl

(akxy )

pi(xua)xlc)

Done Ip(0.0k)1 <2e pour k assez Brand.

=1 Grace I

on pelar introduire un oplrateur 1inare

hl" tei que e(xiy) * <Tx,Y 5F l i ir

Ii

a t eglt de prouver que T est un opfreteur brnaj

que T(BE)

est

bern

dans F I . Graze au eorollaire 11A fi suffit d fizer yeF et de tanntrer <T(Es),y> est bern1 ; ceei r4sutte de fi).

fue

r,

1)

on 2

<Ay - A(K0 +>,), xn-x0 -x> TL

et ar.L Azni x> tn 2Axr): C(x)


qut i4 1

avec en E xn-xo 11 i t C (i:) ilA(x0+x) U (i t II y ii ) (en 5uppesalt ein d5duit du 2) IL5

Yr,)

gut !bl.xn ll rste bota - AbsuTde. que x x

SuFpcisz,ns qu ' U existe une cuita (x rt) cl.o. 1)(A) tenla On cho:_siL r>0 tel que B.(zo l r)enenvt(A).

d< U ->

Denc si X 5 E avec Ilxi<r, on peur_ trire ta el V !, y i E D(A)

. t iyi avec t i >0 Vi, i t i =1. et x o +x 7 i1 i1 i (nen entendu

C. ,y et m 1

dOpendent

de

i.). On

a
13

<Ax n -Ay. x

- y.> >0 nI

e.luen!: il vient .-y,>. Paz eenl y...x e t done t.<,U 11 -v- > > t. <A 1, i n 1 1. n * n 1. a <la n, . . x1 -x -x> > I t. <Ay,,x -5, > z 11 L i. t L 1

u
1- 1 '

zc> .1 e !Lea 1 1- e: (x) < U n> n n

L_J

avec e n

Pp 11xn-x0

II C(x) =c. aAy. W( 1 + x iinri ). et iI "

111

1:4

.51 3) Set %E Int D(A), On =urce ee=e la question 1) qu i n. existe deux constantes 11>0 et C telles que II f II 4C Vx E D(N.) arce x-wo N <Ft et f C Ax.

yoixi , de corte que pour cheque x e eP, i1.71 Pour xE 2.1 en pose T x gi i.ti
T x converge vers une limite. DI aprls le therlme de Banach-5teinheu5. U. existe n Una constante C talla que iT12X I,

Cx

RP

1.1> VxE ."


LP ,

n choisissant convenablentnt x on voit que acjf,P1 ee gola

Nthodc (ii), V5-vitiens que le graphe de T est terral. Soit kxn)


une suite de F. avec Kn x et Tx n I. f. Oa a <Tx n-T5r o x -y> <f-Ty 1 x-y>>0 VyeE.

O et

la licite

Chol.s U:sant y

+ GZ avec t EIR n E! on v i.t que

MIUDJ 1(r-9) est rer=35 ; or 1) Clia -1.u.7..pc12 d'abOrd que T<Y,) est fernl, facts T -1 (1 T-1 (TO) . 11 + E! (T) . Inverse=ent supposons que hl +11(11) est ferm. Cin=ie T est surjeetif on a - s le tblorZm de l'applicetion Ouverte cm sait que C. (H+N(T)) 4T(M). D'apr e

C (11 + IT(T)) est ouvett, et ene TM

ezt ferm

2) On sait (voir exercce Xl. ) TIC Si Nl est Ury. JOUS-espace vectoriel

ferrn et si N est un soca-espane vecteriel de dirdensiori firsie, ah-11-s ;1.+N est t'erut.

D'aprZs. le thorlme de l'applicati_en lavarte i1 existe c:>0 tel 'oit (en, la base canonique de O, T(DE)De 5F. ,D
e = (0,0, , 0, II ,0 (n)

est-a-dire
)

e x iste u e E. avec iiI

Eic Len flux:r(u

=e posQ nL n Celn sdrie est

Ztant
1jen CODvergente et on vrifiJI Illeilewnr quo

rpoiliJ

la gdesLiart,

(11.11..tte

ccinsidrur T actuffle un c.plYa.tEur d

E sur R(T), on pieut

toujours suppDser qt'c T est Bwrzeo"1:f. Raisorinuns par i l .absyrde tent qu


111LZ n

Ruppo5ons ciu f n exi-5te un suite (x n 5 de E


1

fi

=1

et

II lir, II 7

ft

<-. ft

D'aprl.s le thrZr& de 1 1 11ppliCaCiOD OUVWCte in existe une Colnlante c>") 1 tell c:,ue TOL ) =t 1SF . Ce.11-ne Tx <E n ni Tx .E.Ty n n et 1.ztire x Ori peor br.c 1 z E
1
C

existe y E E tel que ify <

y -+ z ,p-Jec z E nen. n

1! -

11

0_ e.

O.

Paf surte

.
I ni
1

D i autre :levet es!:. (pui.sque

'oO

ft

<1 * lyri [
TE

I. 11 U:1'11 0 EL

n1

C.

ZAr les nrn-i-:Pq E II ce

I sont U'1liv 1 EntrA ri27.`

ner) <c..) +

LiT212 Solt dlabotd T'O ; ore sait (unir ciprollaire 11.6) qiurr2 T-1 EX .CF,E) 'Supposons que UE,C (E,F) eaet op .crit I+ u II < 11 tbrierons que T + ; en

C I+ (c -1 .0)
tv:Ljectif p1. . 2 5que It-1 < (nezsquenne

et il est bien connu que 1 +

O) SE

faene u thEurtne de 71Inilit fixe de Banach, volt thor I ine 51 . 7 ). maintenant Ore sait (gr3ce au t5-icirCrtot II. 11) que R(r) et fg:rtt6 ken un projecteIr peor

et atImet un suppl.-mr.la.it tped-Dlique sea..rks F. Soit 13 1F contirsu i t r Qpratcur ? T

est bilectif de E sur R(T) et par COnslquent

1L i

applisuer ce qui pre:1-1.e. EOit ILIE.r(E,F) avec II U II <1 ; E

l'en elw:isit

assl prt 1 "cFp.1=tEur (13 T+ FU)

R(T) est enwra inversible et Qn plit

rE

introduire (1T + pu)-1 Ill&nent

(1'.(7),F4 , . On pose S 0 (PT+1 1 1J)-1 P. II est

7,E) et que S(1 +U) w IE. clair que SEX(1

+.0

1) On considere l'espace Je Baruch E =EPNI(T) et la surjectien carien iglIP

n :E #.1 de

surte que II 11x11_ I dist (x, ( r)) V'KE E.

On faetorise T sous la forme T =T 4

avec TEZ(E,P) et T itjcetir, Biec

5Gr R(T) =R(7) et done R(T) ferina 4= R(T) ierm.

Gr2ce au corollaire 11.6 en voit. que


R(T) ferrs& . 1= 1 9C tel que 11)1 1 < et trivislement i=*

II 'se y

91: tel que

< CrIrly:11 VxcE,

E, E
r
1

* 9C. tel que dist (xI MT)) <C IL1'11.11 VY.E

CI1 .I
D'ppre.5

L'op&cataur

F e5;.:

ersairtra et curiecti f.

tliCorbte de. l I cyplizntion euverte

CZ:1 1.5e.e une etnstint-2 C te11.2

IYIEF 9%)C Ei,


et Soit uEEE1 et n xL 1 Cll f

:,:.::eE2 avec Tilll I: 72%2 7 f

II xi fi + S 1c2 II < C II t II .

.-Tiu. 11 exisi:e done xIEEIL tal que Tlx1 0 1r1u ato pose 1 1 par sk:ite ditr,u,11(TI)) (u-3:1)11<ellIluTI

vuEE1. en

applique Llore L'exercice 11.14.

11 776 . 1
rateur T :C

soit U la surjeczien eDnonique de E 51.4 E/L. On corsidl -ue l'op&EIL dfini par Tx=itx pour xEEG. en a dist(w,N(T))

-diz%(x,Gnz)

dist(x,1.) C5Tx11 VxEC.

- fl(G) est ferme. Done Il en rsulte (voir exercice 11.14) que R(T) .

irlquen =C+L

est ferai.

Ou rappelle que N(A 1) Soient 12 e N(A)

-7 R(A) on C191,(10tV) [1 2 YteR,

et vCD(A) ;

<111(WitV),41 a*Vi

D'oil ion dIcluir que <Avpu>iL O et dont u E FI(A) . 2) D(A) muni


de

la norme du sraphe ese un lsnach k(A) muni de la ore :DCA) s R(A) les 11 5-11 Deb1511

indulte par E' est un Banact9, Voildrateur

du thSorlme de l'applicaton nuverte. Done il knia:e une constante C te1le que vric R(A), 3v E D(A) avec Av r f et v D(A) <C P f II ; en particalier 110 < Cif Solt uE D(A) ; out appliquc ce qui prc'e.de f vcD(A) te1 que ku Av et Ti existe alnr5 1 on a 4-9

< CP. Au . Carne .r-VEN(A)1(A) II Av li


Piler

<AU,U> =<AV,U> < AVV>

;70- ClAu

FYY.1.1] P, lin <:ibt.inkir.a dcw: :as : (a) ir 1 =Ra e: R(A) k%(f) ; ceo G.) 1.(a)* 1 alors A(A) ..{01 et R(A) E. 2) C(K) n'esE pas ferreC ; !linn on dduirait du thUrInie du craptle feirT.IC que-A est "'liorna. Cr A u.i .esw pa.s hornt piuisque f 3) D(A ) tu E E i pas continua,
1

J 3 3
3

<u t a>.- o) et A u = u VuED(A ).

4) N(A) -ie {O} at k(A ) ... &CE* ; <u,a>= O.

5) 'Dn a R(AYL (0) eL R(A) 12 ina (mur que NIE) est dense dans E vor ezercice 1.61. Drte. nSir )4Z(ish) Dans le cas ii) I N w 5UA.* )

*1 1-1(A) 1111:MA h

6) On wat que si A n'est pas ferm6 il pea: $e produire que 1.1(A)91R(d5:1r)-1

iEm
1) I1 Cst
Snit elair que D11) est dense dans E ; vrifiims que A est fum.
f

(u3) urce suite

de D(A) telle que ui --I u dans E et Au j

dans E.

IL en rru1ee qua
Ir / k
n u n Nen (, =1) . Vn. (j --+ d)

1
ne,ne nu n

ntri --4, f a TI

. 11 Vn. ;

par suite WE D(A) et Au = f. * D(A ) (vn) E

2)

(n vri ) E Lsa l

A tv =

vil )

. et T.R7 *-) = ea -

j,:
r-

1) 2)

On a DO

* * * * (ve G t ; T vED(A )1 et & v = A T v YvED(B

Si D(A) 94 et T O, al nrs 5 n ' esr pRz feral. An effitt

itn1 peut

rre.uvar

5, u, avec ' te11 1 1 que u une suite (u ) dans D(A. n

uy D(ti) ;

mora ;S un

O,

21411-1 n1

D(11)..

1) 'Trivial 2) Oil sait


(v.alr ectoliaire 11.6 ) que T-I eZ(F,E). D ' autre par

an

T-1 41s et T T-1 y

Done
T * T (T-1 )

ee

(`r-1)

Far suite

est

hijectif

* et (T )-I I (T-1 )

FIT.27 I On

a
* (T f 5up { f,x> - 449(x)}

xEE
sup 1<f,y> liER(T) - 4()91

Inf

11(y) +1(7)i

YEF
C ( Y) -<f,Y> R(T)

(y).

LI
Cace au thorlmt I.LI
obticut *(

* * g (T f)

g) (-g)1.

OT

)0 Si
(g)= +=I si

flgE RO')

f+g, Rr)

par s uite
* *
(T E) =

Hin dr f.asiEN(T )

(S) 0 Min a 55 (1-11). liEKT )

41

CHAPITRE 111

TOPOLOGM FAIBLES, ESPACES RfLEX1FSh ESPACES SPARABLES, EZPACES UNFORMMENT

conuxEs.

Efff..-T Soit E un ewprzee a Irmach et hoit Ar:1 wn sous-enserte de E,


ecrpar.t rc;ur la topolo3;e muntrer que A est herni.

sciit

E un Csp2CC de

Ballach

icit (x )
O pose

une suite

de E 'cella que

rrur 1,1

tcralnit laibi

Muntr.t.r

pwur

EaLble cr(Ll,E 1 ).

1 1 1

iLJUIJ Suit E uu espaec <1.2 lanza. Schit AcE un cit6-enl-e=ble. 12.mx,ImwMcntrer lile La Ismeture de A 1,1Jul7 13 topoIo3ie forre nt pour la to?olGzOn e(2,E1 ) coincident.

= .5Dit
n

E un

.11ce de !ancLth et suiit (xn) une Sui ts c"-E

q"

prur la toppline faib7 o(E.I.11 ). i: yontner

un, ;h _
y E nenv (U Z yn

suite (yn)

r telle que

1
1

(a)
et

vn

(h) 25 montrer
( a) (1.1)

rcirtrnment.

existe une suite


( 11 EE Qcinv , L.11

) de E telle que

V al

})

x fOrtethEnt.

5711.71 Soit E

un -J.space vic Bauach et

Eczt ur 514-ensemble de E,

a
1

compaot pour La topplogie ferlu. Sois: (ya) erbn $117LC t1 K tulle que x pour cr(E.E1 ). Mntrar que .nt, x forteml

abnurdel. ton pourra Taisonner par l' y

Soit X un espacie tz)polosique et scit E un espace de 8n3ch, Soienf u,v :X 4 E deux appli(ationa contines de X valeurli dares ), la topologia faible o(E E' 1)Montrer que ll applitation dais E mun de la topologie 1* u(K)+Y(x) est continuo de X valeur3 c(E,E r ). runi de

z l e applcation 2) Soit a:X .--> P. une foncLion continue. Moritrer qu, x +-4 a(x)u(x) est confirme de X a valeurs dans E muLi de la rop5losie faible

. .5 Sait E un espace de Beiutr.h. Sot AcE uft sous-ansmbl e fera 110". la tcpu.pnie feib Le cy(EX), scIlt 11:=E un r..ensevsarlite cnupkntt 13, 1.1:1.- la topologia fAible c(E,11 ). 1) mourer que A+ ct fezm piJuv 2) on svprose de plus sue A et fa;ble cl(S,V).

I I I I I1 I I I I I I1

9rdtt carvexcs, ruln irdes at dsjointw.

nutrer qu t ii existe un hyperplen farmc spdrent et 2 au sens strirt.

Arr -

a o:

vurnn

Soit E un espace de Isoach dd dmersien infirlle. en se przlpoae

de mentrer que la topologia faible o(E,E4 ) n' est pat. , meLriu.blei no ras.nnnt ----par l'absurda et ore suppose qut il existe unt c(::,y) sur E telle que

1e topologie assocU.e co:ndde avec i topolng:e faible c(EX). 1) Four tot entec 1E11 ! on designe par V topologie faible o(E,E I ) tel que V w{xeE ; d(x,9). . k Montrer qu'U existe une suite (f ) de E' t'elle qii tour n
k

nu voLsinaga d'; O pOUY la

"

S I dle:q5-42 ICIOG

1 1 1

forrr d'une eombinaison lineire fnie des f .


il

(On pourra utiliser le lemme ILI-21. 2) En ddulre que E' est de dimeLlsion finie. (On Pourra appliquer le lemme de Eairt come a l'exereiLe 1.51. 3) Conclure. 4) rrouvcr par une mthQda smilaire que la topnlogie fpbie * 0(E' p E) n est pes mtrisablc.

[Tnr.Y1 Soit E un espace de Banach ; soft M un scusespace vectoriel

Ce E et soit Mentrzr qu"i1. existe go M tel [Luye lar gEM


fo = u

ufo --ge l.

On Sto lit* ce rlsultat par deux mthodes 1) en util;.s1;nt la t1 arene 2) en uttUsunt la topolnie Is ible * o(1',E).

r1777.11] Soietr C et F deux especes d Sunach. Sit TZ(E,F), te anrie t que T E-C(F1 ,V). liontrer que T est cenfzinu de F I ouni de le topoIugie d(F P ,F1 a velmurz dans E' mun de 1v Lopulogit 1(El ,E).

Soit E un espete de Banach et soit A:E

El une spplicaton

=notarle (veir l'exercice 11.6). On srpoae que pcu: tour n,ye E l'applicliricen ten 1-4 <A(w+ty),y> est conrinue en t =O. Montrer que A est continu de E fort dans E' zul de la topedogie o(E/ ,E).

71.1.11] S.rlit E un esNize de Banach et soit Ko e E. Scit t :E


une foiletion convexa k +

1) mpntter que les proprQuis suivantys sent 6quivalentes 3[1,>0., 111.:~ tels que W(m) t sifxe E. avec Ox-x0 11nR,

(B)

iitn
fe E' f U-1-x

{ip

<1,7% >1

2) Moyenn3nt l'hypothhse (A) ou (B) prouver que * Int {tp (f)

>I

est atteint.

[On pou:ra utiliser la topo!ogie v(E',E), Pu bien le th.lorm.l. 1.111. fuelle est la valeur de ce', !DI

Soft E un espace de Banach.


On pos

(xn) une suite de E et xE

Kft I. c.nv 57 -1 N.} . ron 1) Ore supposr.: qua Ir,


n

poner la topolvgie
411

Yontri9r 11%a

n=1 Kn

R(11. ui (xn)

2) Ciu suppeae que E est r.Wextf. 14->ntrtr que, est bornIe et si

in.

aler51 xn

x your la topPlot'IP Q(E,V).

r-

E17/71:1 7-11 L1
dices. Gn

Soit E un :pace de Banach rEfiexif et soit 1 ur. ensenble de E et un sovs enj" ,17-5 -P 17 . Jt ll

donne un sous-ensemble (fa.

de E. Sott >1 5 O. Montrer que les deux p7oprits suivcnte.y. sont 1.1uivalentcs (A) 11 existe 1: E E avec bid' On a ,que a

1e ifd iEJ jEJ 1

pour toute partie finie 3c:I et toute famille de rels Compartir aux exereiees 1.10, 1,11 et zu 1e~rse 111.x.

igaryceP:tra

d'une rwsure

:ni

convexe.

Soit E un espace de Banich rflexif et sot 1:c=E eonvexe, ferm et bornl. K est done eompeet pour la topclogie e(E,EI) et on considIre l'espaee F s C(K) buril de 50 notase usuelle. On r'I"Ke

1 1 1 1 1 1 1 1 1 1 1

C ri aves II u ll. I

F Cn

5uppose que >r>O i.e.

> O Vu C(1:1) , u> O sur K.


Montrer qu'il existe x0 EIC (migue tecle que - (1 ) <f,>

len pourra commeneer par montrer qu'1 existe encuite l'aide de Hahn-Banach que xo e K) .

xr.t e E varifiant (1) ec prouver

Soit E un espaee de Banaeh, 1) Soit ',"fas)

quito de E. On urprise que pvir tenvt. x= I, <f ,x> n

converge ver: une lnte. Montrer riu ,1), 2) On suppu.se mantanint que E. est rflexif. Isait (2 ) ene wlitit de L n talle que pour tout f e E' <fokri > converse ver une limite. Mantrer qu'il existe xE E tal que x, x pour la topolollia faible oU p E'). .:onclus ion de L) existe fEEI tele que fas f patri la tepJlogie faible*

3) Construire un example d'espeee E non rflexif to.nbe en dfaur.

(On pourra prendra .1.111 cD (voir exercices d1 nap'irre 51I) et Itri o

--,1,0,0.., ):. (n)

LILI . L 7
) SOit.

(2 1) une suite d' lment s de .t1) avee I p

On sLippos que

n' --h x pour la topolo8Le cr ( tP,tr ), Mentrer que Tri-co


a) (X ) est born dans 7.

b) x. ---.m

pour taut

phri. note

et

2) Rci.prQquenent, joit (21) une suite d 4 L''ZISWerit On suppase que : a) b) n ) est bical dans x, Y. pOur toult 1 el quft xri trigss

de LP 2.41, ee I <p.".

1171-m

,E MOntVer que p.

el

x ppm' la tOp010.gie C(2.11,1.111 ).

7f;

H7 ,

c:I.aque entier Ts>

1 011

pose

(n) . : e dWrIE F pout LA trIplogie 12(2,Z151 ) AVEC 1) ,.....ntrer ve en. p


J<FIZII'.

. g! (01 k) ciai coDvere 2) l'entrar qu'U n'eximA .auTu..5.2 sowl-ruitt c!ztrn'J.I dans 1.
t}r3 MI

ta topolcie

G (11 >

P.7)

CU Un egsemple d'udN.ce de :ffiani.L.zh. 'e: et

(r. )

LI

que Uf

Yn

t selle ciue

(fa)

ne poss7edu aucune Eiyur-s..1:te corlixa.lietifyu la cour,c:itI

topellezie LAME. 44 B
11.!Dor }g

E'

?
d

[OrL pour:a chnibir E a

iirl -TE Soient 1 . 7 =1.11 et l'-1Z9 avec i<p<9:. et 1 Zq<une functian contilukt terla qua la(t)1'(Clu;: )111 YteE. Etart di:511A .`(xl h 1 :2 y cn p' 5(5: L ),a(x 2 )*.1) Iloutrar que Ax k dares es (.1ort).. tt que li sppliraton ). ) t9

a :11

s.

1L

zontinue de

p (fel-z -;

2) M.untrer que si (mil) est une suite de 22 tecle que faible u(113,1,13 ), eiws Axn pclur te.polee

x rour la 1

G(2.421 ,0 ).

3) En dduire que A e:st contjnua 41e B mufli d e la topolni 13 {E,E I ) E


valeurs dais F muni de la't,apollle Er(F,F 1 ).

Sit E un esillice de. Banach.

1) 1.1putuer qu' A existe un es pace tppoIesique wmpact K L une iscalltrie


de E dans C(K) muni de sa norme usvelle. (i)E poxtra choisir

purd de la topolazie feibIe

cr(E',E)j,

2) fin uppos de plus E sqparable Yontrer qu'il existe una jsomtrje

e E dans R .

Soit

un 2....p.ace da Sanach
App.21 'AUX

loin

) und suite borne

dz; r 11 . 1:Lintrer direetemnt, se.r.s JZJiire

prprT5t.;s 4e d7 L1,:tris,111j11 . t

qu'U e511,1te nn2 smus-suitie (f, ) 'k

colverle yOUr la tpplawgie

faibl

*
un proeUl de luir-e 21g,,walle].

L111,22 I Soit E un .es pac de Banach cht dinensiDn itfinle 17-rifant l'upo
des hypthses suivantes a) E' est sparable, b) E est rffleldf, Montrer qu r il e::iste une lliE2 (% ) dans E Lena que Ox pour lz topolnin faible er(F,E'). n

4, 1

VI% et

17175 1

La dmJnstratinn duNth5name 11,15 est ecilsiareblement


X

si on suppose dC plus que

est rfflellif. Pburgund

lExaminer

(b) rt (a)].

60

Soft t un e5.pane de Banneh. On se propse de rWDDntter le 1.M.-

thle II1,25 1 , el eilit-.5-dre qu lez1 rbreprihjs su/ventes sent &quivalentes (A) El est rApanlbIe.
1 ), (b) DF est m'trisablc* pour la tr>rologic faiblu e(1:.1:

Pour l l implication (A)


yna III.?5. Pour (3)

(B) oei pourra s l inspiler de 1a dll.TQnstratien du tho-

(A) on preces corma . 5uit, tr(EX),

d(x,y) une mtrique dfinia wr TI qui induisa La topolu E On pose 1,11 {}r.G d(x0) <--r

Int V en voisinagd. de C peor u(E,E 1 ), de la formt n

1111 - 1xEE ;
avzc f. >0, TI

1<f, >

< . SciL D

0 1d

tv

cEl fini,et tal que V

- Lj

at 591.0 F l l espa;ze

yuntouiel err,endT pat . I. on va ineintrur Lu.r

clt rs rt e Ir.r,s E' piDur la h - ope-

logie !zrte. Gro ritilonn2 par l'alisurdf, ot c,TL uppge


1) r.kyntrer <tu'

zixisLu CE F." EL f CE! tala que


o <,.E>d O

<Z,,,f0 > >

vfe F

Cr. ni: "1.

2) Soit
W- ixER . x>1 < 1 /2] P 1 <z Ilontier flul il existe n >1 tel que V =1Z. ilo e 3) ri:01.1wr qui n exi.gte xi

tel tel .c iva wilc.

1<f p:g. > - 4l p f>1 <t,...

u.

1<foixi > - <4 i f ei >1 < 1/2.


4) En d duire que l e v o

que

<for x1 > > 1/2,.

S) Conclure.
Soit. K un cspace trique ceitipmt nen rilduit . un nembre iini

Fli1.+ 25

de pnints.

~~

hontrer gut l'espacie C(K) muni de sa norme usuelle n i ost pos releNif. (IntroJuire une suite (a ) de points de K Lene que an n b. Conidjrarla forra.. linairc f(u)= --- u(a n), u..*1 21 a et n 1 6 a Vii. n
et sliosnirer de

l'exereice

j 111.261 Soit F un espaco semble dense de


5,

de

Banach spacable et soLt (a te) ur sous-en-

On considlre l'eprateur assoeie Tr..=


I
XJ1.4

linaire T ;

F qui xii , (xx n -) 1' 2. 1 - "n"

1 1 1 1

1) Montrer que T est un oprateur borr suricecif.


Datas la

suite on suppose de plus que Ir est de dimensin infine et que

F' est sgparable, 2) Montrer qua T n'admet pes d'nverse droite. [Oto p;.purre utiliser ler rsultats du prohibe 3) EE, od ure que nen pl mimet pa:, de suppljwanteire tnolo2,:que d:1-1 A) DAt.Proiler 1 .

1
1 1 1 1 1

prE7277 Soir, lI un @space de llanalb sCprible de novuu II


norme guivalento

Cr_ eZ 1 k;igne

afussi par 1 II l lizrme duele sur E'. On SE, ilropobe de constru-re sur E wie

a II E, si:rete:men: convexe t e: dont la norme dua12 12.st ausbi

suietement convINe.

lu it (a. )c:3 un Bous-ensemble n E ?


Pour

dense de B . Sois (b n) E

.L.

t un cill.J1-Ind2T-

Me dense dans 3 pour 13 Lopologie 0(E',E). Pourved ur tel ensemble existeE4

f E E'

on pose I 11 = f 2 4. n.1 1 < f an o 2

liz

1)Montrer que 1 B 1 est une nOrme quivaIente

II I.

2)huntrer que la norme I 11 1 es t strittement convexe, [On pourra utilice; l'exereice 1.25].

1 1 1 1 1

1
PouT xE E on pos 1-1<b .2c> 12}112

ax[12{1i)(111+ (al) OmU l


Sup < f a f al 141

uml 21

3) Montrer que O 0 2 est une norme zerittemtnt convexe lquivannte

R fl,

4) hantrtr q ue la norme duale de 11 h

est aussi strit remera corwewc.

ton peurra utilser un Deultat de l'exercce 1.221, S) Proponer une autre approche a 1 1 aide des rsultats du PrJ1W-Ime

FIDI711
exeralee 1.1,

un espata de Banach vn.lrormlmert cnriveK0, Cr dE;ailne u,r F

l'applictian de dua1it6 (nultvoqu) da E darla E' ; vcir corelire 1.3 tt

!-Sontrer que

pCnIT t01.1.1: f e

E' t1 e-wiziza xe E talicluig

'''t>1)

rfri72 -71 Scdt E


1) Mantrer que

un espace de ilanbc17 uni.1111-m&nent convexa.

lp, 0,
II 2 1% 2 z

VI> O, 38>O tel que

11 112 +11 112 2 7 1 :

Vx 17 1

pvec

y.

I! y1'

;1.2.:

>

rOn pourr a nal sonnar par 1'ah suvde ,

2) Whne q uest hin si l'on remplace 0 0 2 par II !II:1 vet ]

Soit E un -espata de Banach muni de La nnrm2 d II . en SUPpball


qUI iL

existe sur E une norma 1 1 unilonmkaGar nonyexe

. 5

Mantrer qu, pcour tem: k> I, i1 existe una norme U ii uni.E.pr71&-,Ent: CV7I.V17:-E tall que

< 111x111
[Ort paurr poser 111%111 2 - 112c II +
cite 111.291. Applnation F E=P",

kIIx

V>: e E

x1 2 yac F.1- >0 as 1112. pvcit t appliquer 1 1 excr-

1
11L31
5 ment cenvcxe. Montrer que Soit E un espace de Blloach uniforwl

Ve>0, Vae2(, 3S> O tel que I tx e -thril YtE U1,1-113, (Si t <t.
2

I - 45 11y11 <1 et

Vx,yEE av.ce 11x11 <t + (11-t iy

Dn pouTra crira

)*

Proizetion eur un come= remad dans un espace unifolltent


COmexe. Soit E un etsplee de Eandch unifornIment eonvexe et sis,t C c E elDnyLxe, tema ct non ~~1) Montrer que t'out. tout xe E

YEC

est

etteint en un point unigue de C notl r c2, 2) MzAltrcr L'Ud ulule surl minimiszntn (ya) converge iorteme7a vers Pcx. 3) Mott.er ve repl'It.AHI,1 ?cl: est czntinut Qt E rort dans E ro 4a

4) Plus nrUlsilm2nt, wo....trer que 1 1 c, est uniformOxvnt e3ntin-de sur /es bornSs de E. [0 .19 poul-ra Soit 43 I t ewercice 111.29.].

1-.P,fml une Zunction cQrsexe, s.c.i., tpt

5) ivr .ontrer que pour tout xe E et tout entier n> 1 InE{ollx-yil'+1.49(y)} )EE
est atteint en un pnint unquE unt yn .

6) Mlyntrer que yu

Pcx u

c -1511) .

CHAPHRE

DreprZa le th1orlme (lit hanseh-Steinhaus il suffit e vIrifier ..our la topoque pour tout Eer l'ensemble 1:(A) est born1. Or f est continua 1.

cgie Ci(E,E P ) et A eb compar:t pour la topologie o(E,E1 ). Done f(A) est compect
et par suite bornl.

II faut v&rifier que pour tout f V <f 1 on> <f 1 n> <f,x > i <f,x > <f i x>

fix >

1) Or- pose G

(x,. 1 r. \ Contoie x

poar la toieloi,,f..r. tr(E,E 1 :.

-a(r. V) CunviLincl les fermItufts n dilduit que x E G vn. D'autra part, etzmiC e n n de C pour la Lopulozie faibla HM') ist pour La toilik,ilogie fccr-le n (voir exereice Par suite xeIn Vn. Done, x fortetene.
' krile '11 011 E U t COn ti:1.; ir f..

(Y.5

t211e que yneCn Vn et

2) Orz pos e Kr, I conv U j Ix i )) 1 de surte qua (11n) est uno suite. eto. de convexes. Soit on adist(x,V.n) ; riontrons que p r IO. En effet, supposoni, p2.r
l'abaurae que p;,11>0. pour tout n. 0n 17eut alers 51parer ru sens largo le wr-

vare ouvert B(x,p) et le convexe E* O tel que <f.xspy> < <fxn> Dteia il rsulte que <f,x> 1111

K. Par conslirwent, O. existe ZO.T...1 ,

Yy5, E avec ryll <1.

<E,x> - absurda.

1 .2T

LaL2.] 1) s it x (A+B). O n Ja criRLruire W voisinC de O piour la topoI


0(E,E1 ) qut

(3.:+14.)
Povr tou4 ylEE

V( y) voisinne ccinveme de 0 tel que

+ V (y)) (tal- A-1-1, est ferm.1., :.tit xg it+y).


On

n (A-Fy)

V(v))
-

a col= E ed.t coppact 11 existe 1 fini cel que BC,I, (y. ---V(y.)) ave pQ5t Ii 1 IR Yj) ' Alcrs (y+W)n LL.u1) r . En mED:It supposcms pz;
11 + w E

qu'il c2LIsza

W tz) que

(A; 12) .

Llile 1 exista Lel tal qi]

Coi. 2 1,7(y i) ese conveze, on en Uclut qu'll cdste Id.' 5 Inyi) t411 que x # w 4 E
rar suite (1L; .1 '9(y i)) n (A+yi) f - all%uvd E

Rtmarqu'l Lig. ..roque W nst sper2biz et A est iwn.15 on pzut utilistr de5 -L2 czr lq roperiqpie leible re3LT.dint4 111AX bDrZuites piur pyzuver out A. 411 est fv1. ns ust thUr'qn 111,25 1 )- La raisonnement devient alors plus

1 1 1 1 1 1
1 1 1 1 e

5.imp1 a . Ln u z7.f et soit Xn an .bri evor

x pout CF(E,,17), an e A .0 b a e

xZ(A-kr1). Cerae 8 es.t E0ibl2m=mt ncmpant Qn peUE 5421>pC15.er que pour 1-1(E," : 1') evec liE fenal on en ?.1.duit, que Dit:i u
X b

nk

pouz Q(E,E i )

aomwe A est faiblemenc

x -be ti

et par suite xE

2) D".pipts WE qui ptl.e.Ide (A-II) est taiblument ferm et done aussi fortement fenme. Or F e.at a] ora sparer {12-convexe compact-ct au sln 1 strict. b=invexe Eermi

s4111.8 1) Canee 1'k est un voisin.rige de O pour tr(F.,F- 1 ) (voir proposition II1.4) que Vk est de la forme V iNEE ; 1<f,x.>1 < k k o c k >0 et F est un sous-enser.ble tni de E'. Par suite l'enserible F 1.0 U] k est dnombrah1e. Montrons qua tout gEE I s'crit cornme combneison lina3re finie d'l1nents de P. Soit geEl fix et soit V ...N:CE ; I <g,x.> I < 11. Creme V 1st un vi.yi2.1nrle de CI pour la topologLe er(E,E'), a fortiori Vk cV U Si xE E est tel. que <f.x>l 0 VrEF tx1V VtER ; done <g,x). E O. 0;1 rrduit du I
SM

peut tou,nurs suppoicr

k te1 que (x E E

d(.,1,0)

V et

k'

alors tu.E

vt

et par suit.1

ra 11.1.2 que g osr. co:7.ibirifilson 21 .:


En eflkle,
ir I

LI P.,

2) On. raisor.re exaetecoent co=e 2c2ssaireme,lt 3) Si din: E' <la, alorz n...L

a una injectin carlortiqte 3 de E dans F.H . 4) On ctili.se ici le lemma auivant f,:t, >. O 'el ] ' 1' i 4-7. f.>>. O:. n I Atara il. existe des rels Ii i .,..1. 2 ,...Xn tela que yu 1 ./..x.. . (ce 1c- ..e . est 1..;.;,.. i.1 1 i Soient x l ,x21,,xu, yEE tele qua [fe E' ;
5 querice

faene du lee 211.2).

1) Appliquer le thlorlme 1.11 avec ID(x) <f oi x> 4 I B (x) et E

1(50 -1 11(x).

u
r-;

2) On utilise le fait que .A.) B est compaet pour la tepologie Eable* EI 1 b) I4 est fenal poiSc la topolcgie faible c(E r .E) (pourcruci 1).

n luffjt le mon t rer civi l si unL suite (x ) de E converge forten taent vera x . alors Axr por Le. <Axn,y> -+ iskx,,y> VyiE E.

SupposQns, par l i absurcle, qu'U existe yEE tel que <>.3:11 y>-f-4, <Ax,y>, D'aprls l'exereica 11.6 on 5ait d6j que (A.x ) Esst born.. On pet.t done n extraire une sous-suite talle que <Ax Arpl wonotonie de A il nk A la liTaite on cbt:ent t.2.4 e <A(x+ty),)..> b' al de.ditIE (dsLins,uer ley cap t >0 et. r <O) O. .1. di <Jkx,y> - ahstirde. -A(x+ty), ien -x-ty> O. <Az t y>

xnk

177177
(r) ;IR !: M EI * Inversement an suppese (S) et on note 4(f) 1-4 (1) <foco > Montrr.ns quin existe drs constantes k>0 eC C telles que 1) L'hypu-aizze 1A)

(1)

ia -

vf

.
que 1,(0)<- (voir

Quitte faire une translation pn pe.ut Su pose 1.9). On fize u>1(0) ; grrice p(g) a

1`hypotiplac (E) i1 existe r>0 tel que


Nig e'

Ei

avec

II z II

r.

Etent don:LA:. f E E 1 ave..1. 11f II >r on 1(EZ)<til(f) (1-01.1(0) avec L et corne t II f II = t II vient a - 41(0 Trft Unf) -9(-0)). Coci conduit (I). En pissant de (1) la relation conjuple on obtient. (A).

2) La fonetion

est COnVL:Xe 5.C.i. mur la topolrgio o(E I ,F). L'hypo1 1 onsemble if ; 1,11(f) <1,1 est boro

tUse (E) entratne que pour tout sute tet .ensetblr

eempeet pour la topoldgie o(E',E) (Erillee au theorlma

111.15). Done Inf 1 est atteint. D i antre part oca a

inf

- stip{ < , x0
far

*ror
(f)} = No) al -CP(X0)

On pourrait auss retrouver ces rsultats en apptiquant le thllorl.re 1.11 ak:x foncitons cine II.]). et Ifx (notar que la fonetion 0 est cuntinue en x o eKer-

1) Fi ut tout p fixls n a x Pin

P Nion. Ala 'jaita (q9land

rr ob-

tient zelt puillue E est faiblement brrn1 (voir th154%rZma IiI.7). D'autne part selt V un vc,isinagc convexo de x po-Jr lc topnlocj.e 11 existe tel nue x esr 'kn>r4. 01 K
WID N et par ralTe

rk

AV' 1.x)

ot'erdsigna la familia dos VOidiAEZet, C.D.WWE dt Y ?:..11. -

(justifier 1). l , t y un yols i nage ouvern de x pour la LOPUlCiU QU'1 1 2) Si : 1 ). On P1151 Klaqc 115. K est eompaet pour la topologia (E,E') (Enest convexa fe,=a born.1 dans un r6flexif - voir eorollaire ITI.19) ; done K est aussi ccall -met. D' nutre part F s 1 l4:; 1:=511 et par state 11 exiEta N tel rus 1 , ;, 1 vs. 1,

2) Supposons par l'absurda quo Alors <e je n > 11~1 <c,a> Y111 11

--- a dans ti pour la tol.olole On censidara 611ment particuliar

..,

).

ce Lis
.a.m.m II III

Ufini par

(n1)

(T7)

(n3)

et do=

tk

.,k > = (-1 ) Aorte

lurnviSe pas qund k

Coraradiction.

3) seit E= 97 de f t ic f dans Y T pour

ril:c

Z.1 Cr'

pny.l. In = J. Suppesel,_5 que


<

la topplozie n;11

miz

F:

pour tclut

E E, On c.J.1151.d'r ..1.1f11 ,1..F.

1 1 11 21. e . ment particuIiet

cor:ma ' la til,usabsurdt. IL n'y


quE, 3 E'

tion 2) et den.:

(-I) k na c.:orivarze pas

pas de
tuni

contladietion avec Le tl-Plorlole 1.11.15 ; ceei montre seulament


III la topol.nie auss. est

Le un espete compact. rwn mOtr.C.mbIe. Di vetrenwe

le fait que E. =

n'est pas sl';parabl (appliqutt le th.1nrme I11.25).

LUI. 19 1
1) Qommancar par vrifiar que si xl
112'u >:

x dans IP (fortY olors

gi

L. que

15j 1 .1l .,111'

Vr1

2) Utinser l'e.xereice III.11. 3) Cr.; l.). l'e:gpant S 1 1:1.25 1 ) il sulf;t de pout


la

torDlosie
de

('e ir thiio-

r)L77 2

la ezintinuit

A suv is sulltc;.1.

t)

Vapplication T :E

Ca) d'finie par

er:0(e). - < r., x > x.E 1 E a t'E Sz, Cu a bien 4 Tx


2) It = RE ,

Sup (rz) teK


est un espata

- x II altrique
CumpaCt l'out la topclit 0(E',E).

IL

exl_see dure

un sous-n6etribl

(t ) dInombrable dense dans K. cLefune Par

On. c n 1 re

1 'applieacon S E

sx
Wrifier qi. II Sx II 9 7 I Ilx

< t lx > <E2.x >

<t ix>

oit (a.) un sous-ensnble dno.ebrablt dtnse de E. Cl extrait ttria. pranib:e ETV5-suite Leila que < f nk F c 1 > crver,e quend k eci
3U S 1

Gn

oztrait unc soiLiz -Bous-suil e ten'? igice <


eigcl usuel de suize diagorsale

El

1 a 2 > con"12:ge .' etC

Par un pro

n Gltient alDr

rtoEIt

), extralte

de

la 51.1ill-q?

telle 1lJe
r .e

VI 1a
E..I nSiZ CI'2 S a T12 3. Vit E E.

Ore ii
r

2 El d dIJ

D' u::

rt'eulte que

ni., coriversi t paur le toptilgi c5(.E I ,E) (Yr5ir

Lr

rL

Lui.227 a) B E

est Tec' etTiseble pfflur La 1 -011.010:;i2 e(E,,V) (voir tFziloribLE sphlre unit por la lopolop e o CE,E 4 ) (quir

Di t autre part O ast adlilreot 1 1


refivirque 2 au Chapitre T11).
b) Carrine diiti

volt consLruire. tl

5eus - Pirape

<ti Cluc

rl-

nexif),

et d,2 dimen_sier. infinic. Cin ap:)11c....e.

F.o le caz; a).

Su.pprsam, par 1 i i.11-.isurde,

COZ> sei= 1- 1171-"ii Mor;

r
r

E= {1.2CC-115'.) ;

egt auzs 4. r6ft.nzif e t Sup f(u) u:n atteint.


uq8-

T.) " autve Fraru #lLp f (u) 'LIE P. F


11(n i 1 = :3, .14

5 ; ea ef :Set Ihf:q., 2111= t'A lac


par le

i et
. .

(IQ 7.1.e

.L1'7 E
a.burJe.

gua E(,])

s1 alor ..(J n'

144) 1: .O

1 .) Szbit yE

17'

fi exssa

t elve <1122

1 -

1.

L
r
L

est densa fiFins - E .a..,. dc,Jaa :.l eniste ti.2 >ti1 t.el que 2 R

I 1 Ity-a - - a II < - ni 2 2 11 4
Pir t''eurve.nr.:e rj coratruit Lne suite ny t
".

t elle que

an

+ 1

1'

I 1-2 + o, l

1 2k-J e tlt

2) 1-0,r l'ab%..urdp. Suppusons qui i1 existe S F.0 (r.31) tg1 ei;ie


ail

.p,pr

eurs, seit (y ) une suite de r telle que II y i! ." 1 n

Va. et y

O pata.

la

topolezie failxl e o(FJ 1 ). Ahrs ly O pour lo topologie taible Er(.2. 1 ,t.1 ). on en dduit (vair prolillsie yii .Tsyn 1- 0 - absurde3) Voir tliZeirloe .4) T
t est d.firt par

) qu Sy

fortrent dans ti, Done

1111.27 [

B E , est un esplice r7 trisuble canpact pour la EDpeltzip. c (E r iE)

11 existe r!'..Qnc un sous-ensemble dlricvn:brable dense pour cene tnpollogie. 1) Cc a Ilf I Iltill ...71 11E11 Vf e 1

2) On po.se if 1 1 11-1

< f,a 1.1 >

p. Remar q ue_ q uo

1 est une norme

(rourquoi ?). I1 a"ait de v-a.3fier que la for2c!liatt f 4h- fif E 2 + 1E1 2 eS t tt ie tement. nVexe
OTI

nointre lisZeent que YtE[0,]1.

f.

EE

vi;<!-c);r:-gi 2 ti11 1 + Es.r tonsGqucul L


Lin

fooctIon f

1E1 2 est sZrictemeTit conihme

que la vont-

1 2 + If1 2 .

3 '11L -1 mLE:03411 qu'a la qi.Lezti.on 2). IrZoter q1.1 si <bn ,x.5. - O iiirt, dL Qr5
X g`O (pOurcwoi 1),

t.

4). on pas
duele - rica

212

2}1 : x> rG

et on 11..1slIna par 1f 1 la nortat

unir zver La norm df ini4

a la questl.on 2) !
I! est sttictement convexn.. or 1 rie la felietion x 11-1- 1; 2' n Z' Appliquanz El { f VgZ" 1h1 21

IL faut iy:' rUler que la fahetion f


44 canw_Ide aves

con '

I'exarcien t + 22 lquestion. 3)) on voit que f II f {II f-11 WE r 19 'I

EI T.autr part on. R er_core 1' identiLE (01. Er, effet ot a

I lifi-(1-0g12 =Sup(<L1+0-r.)gpx> - -111 ci 2 ) x, t -C.1 2 Sup{ <f- ,,y>


';

'1Y121

t done

liltf I- (1-081 2 .4e(1-01f-tp I -Sup{.(tf+ (1-1)5.3:>+ t(-t) <g-s,,y>-41 (i-t) ly1 2 ] 1-112 x,Y
L

Ort conclut gr:Ec au, henzement de varl Pbles


x = bC

- Soilent .E,1E
L

(1 -1)11 et y

; rb fize h1,112 e E P tels que 11 f !I + !hi1 2

IE gil I II z -]12 111 + tlti1 2 Soit te 30,1[ ; on a + 1:tf <ti,'


(

+ 1 thl

(]-01-2 2 1 2

<t'II-1 7 11;4 (1-t)11.:1:!1, szuf si f h 1 et 11.z i.e. i

1. ...cant riliflexif. DigB Aleas


Aa.g*Ti.tr

< ';,x>

rt

tteint en un poln.: y. E ;;

r..

S FIN) Soit FT l'applica dm:. de duaUcl E I Jets


El,.
I T!SQ7.-

ble F' (1) est pon vida icarolleir 1.3

strit &e. 7 t f . Come 1: est ri.I.ELE.m .1, ion ekncanr.i.it. de E dzns >
II" <1,1E>

il

e.xi5te NE E te]. .1jue

(1 est 1'

II 111 = Ilf 2
F(x) Soient

lx.11

et

e: x tels que f l'hl) en a

('E E(x2) AlDrs

1.!..2t i 1.! = I6 x 2 L1 = Il f II et done si X I 96 x

< rG L
DI Rk,ltra par <1,z1 > f ix2 - 11 f 11 2 et donc

lif12 .<fA_ + 2. 11. < V fiI 2 si xl# x

~~

1) Sup?C.5.0115, par (x
), (y ) telles

l' hlis.uvdo,

cr.i.cte

fiC

deuX SUitps

que
II z n II 'QM.,. El yri. 11 >cc, et

CI)
Quitte 1 extrare une

II

111 1 11

>

II

p.

II .+111

Yn

2 I

ou -suite

on

p L eur supposer que 11Kn ri

-4-

etyn

II

-+ 11. 1

rG

obtient

. 1 31 . s .rs

a*b>e o

et - 8 2 + b 2 2 2

($ 4)

2,

Par

suite

bl

x.
X ti

,4

Y TI g
et dans la sute

E
,
E
on dlsittndent

n 11717 n
n

el f m n ily n
+ Q(I) (ici,

Four

assez

Irand

on

lix ' -y 1 11> n

grm par u(:) di'....ews JIs

quantits

pcailimes

.
quaLld n

pu rizaLives fui ex:".51e


o

vera O il l

ce), 17,rAat I ' unfnrime cnnvemit,

>0

tel

que

x i

4 #y ' 4y n rs1
2 M

4.

r
C

n h 11 51 R"

II
1)1 .1prs on a

(1-15)-Fer(1)

+yn a I n2

a: . +Q(1)

ei. 21C49: (11 i5 ) 2 + c(1) -

absurde..

1) Le
corollaire

mnimum e.31
111.20.

atteint

par

tt

lfIexif

et

un

pevt

appliiluer

le

Ll unicit pruvipnt .du f2i1


I ~PHyxfi l tlz

que

est

strictewnt

eonvexe

et

dona

la

fcncti.2n y
(ya)

sttiettment

canvege.

L L E

2)

Soit

une

suite

minimsance

Qn p05.1
corte que

et

d =

51-'y p

de

dn --, d.

yEC On pPuE eztraire une haulsuite (y n ) lene


.que y -lk

faiblement,

alIDTS

zEC el 3 x-yd

cpourrvar,

Par consgquenr 11 vieut x-z x-Ynk d- EK-a. fDrteraent unid:, t.

Ii en rgsulte (vea' propositien IlL30) que de la litaite iDplique que touto Z cisen le rnisonneinent). auite y n

con.verge fortement vera PaX (prl-

3) et 4). Supposeens; par l'absurde, qu'O_ existe l > CI et des suites (x ) , (y ) telles que, n xn r. <111 On a done
Ir X - X it C

yet II <1{, Ir xn-yn II P x +ID en eyn 11 II 2x +y

O et II t'cxn-Pcyn

2 t
ta

"C x +11 a C' 2 | x +1 1,y C n r, .n

1! ynVer consiaquInt (1)

2 obtient
x i2 +

o( 1).

I [I 2 j 'Iba C

Yn - c>11

(I 2 < 711":1Yri Prx114rCYL11 1 2

+ 0(1).

D i a.-nye part 'De pose an x -Px n n et b-P eyn'

de sorte que Ilan-bu t to + o(1) et bart II Cr2c .13 Vezereice 111,29 11 e>z5te e>o tin b + 111 b < -1 2 11 II 2 n c i est--li Lee (2)
y P ix +yn Cn'C DR , 1 , Y. "' -gd z e n

+III n -0, , 7111 - -6 . 2 iy

Combinant (:) et (2) on trouve une contrsdietien. 5) Raisonner mame 1 la questien 1).

6) On a TIA y el+x II 2 + LP(y L )

(3)

n II y-51 ii +ipy)
tiPM Carie; quo (pW0-

En (JE tiisant le fait que

e5 t winarl. par una 101"4 ticin

po s tiou. 1.9) an 'Volt que ( n) reste bcirrie luand n

(Fv.cisez)

On extrz..it une strus-sul te (y . ) talle que y ak z fa iblertent e t 55 E D(1.1.) Ipourquoi ?) On dciv.i.Z de (3) que J zx Done z = Fcx 1511 12
n

wyD(r,p1 cc en5ui re Vy @ Dki,P)

D (1,11) .
Dn

D P autre part, BrIce . (3)

lis sup II y n-x < E y -x


et en. ;.articuIier

Vy C C1 ()

I im s u p yr - DI z-x. . Ot cartela[ gut. y c es t z Eunn,


.

Cc la 1

t (y ) qui . .re tuve rge f 03 tewnt via=5 n

B
II. u fl

Lj 1

CILAP . IUE IV

LES ESPACES 1.9 .

Digns tout ce chspitwe, et sauf Indtatcil esFpate mrlsur3 muni d'unrr mesure On nute U f au licu dt II G LP .

9 d'sgz:e LL

;10ilat a>0 et 11-13.2

B>0.

f(x)- p
A quell.c2 wri] tions n

1111) -1 [1 +
GR )

E T...12 (2) avec

p <es.

1)Z3u.t el-isque 1- ES el n enti:er >I on posE

si
si

ItHrl

jr' 1 M0 ;per que T nt f dais L(a).

1 ri n

2) Soit (2 ) une suve crois5allEe d'en5EmbleS nEsurMg teI .= L sQit k la fonztion ea',:eet'dristique d 4 f dnns I. (2). x T n f dant: 115(M.

que

nn

)iontrer . qul x nE

35 nu.nluer que

LCV.S

On supp.ose que 15:11. Solent 1 11;p<V17 -El

Flonlrer que 1. (1 MCL11(1- ) 3VCC injection certinue 'plus rnZeisement In a

I 9E11 1111 9 q 11E11 Vf E Lq(9,),

.92g.alitl de Wildt .d. Uu tlizer Irit,.

LLV.4 1
1) Soient li ge LP(2) avec I < p<i. Vrifie.r que h(x) trEax[f(x) ,g(x)1 E LP (SI). 2) Soient (f n) et (gn ) deux. suites de If(11) avec 1
. f n 4. f dans I.,1" (r2) et g

cenes que

g dares LP (P).

On pose h is hiatc{f ,.g ). n n n tiontrei; qua hn h flans 1.1(11).


ern.

3) Soit (f ri) une suite de LI(1) avec I C p "nu et sy.it (gin) une Sutr! L (n).
On Suppase gnu f n

f dans LI G.), En #g daos LP (r.).

g p.p. et Pr ri li ca <C.

Montrer

f rign

Sziiont. k fonctinns f.f 2 1.-.*Ek. tene5 11.2 f- E ipi,..1= pcIEC et 1 L DEI Fi le (3t) 17.f - (X) . 1 el.: que k . f .11 1 Pi.

Vi, ave':
fl

I Yontrer 'que f LPG-1,) evet . L L

.1 Pi
II P

(Comencer par le cas k = 2 ; procIder er_suite par rcurrencel ' 2) En dIduire que si f E I" ,a arree 1 p n (r et 1 i", atoes

f E 1.r (n) pour tout Y Comprig d'are p et q. -4- 1 Plus. pri9cisr..:inent, si It'on Icrit 1-1 12 P q
Ilfll

ave,: 0 u C.1 alors

.1. f " f

e, 1 0 Ct- rj j.

1 rr 4"

IV.

Sgiant I

p <4 et

:q<=b, 1 4

1) Montrer que Ll (i1)

nT:(0

est un seis -ensemble dense de LP(n),

2) Montrer que t' ensemble

1.i
est fertn dans LP(f).

(te if(r1) n

Lq

f li g < I}

3) Soit (fn) une suite de 'LP('.2) f Montrer que CEE p et q, 7:91 q . n


r

n Lq(ri) et

ECU E LP (n)

On tuppose que

t dans I,r (1-2) et q ue e f n i q <C. (n) et que f n


r 4 f dans L (n) pour tout t compris entre

G
1

".21 Ort suppose que


Soit fe 17(9). Ileontrer que lin IIfII p = 11E11..

"".

2) Solt fE 1
On suppos e

p4:1

LP(e).

qui il existe une ecl.stinte C telSe que Ilfil 6 C p f 14p</e.,

Montrer que le ( ). 3) Ccnstruire cine feloction fe

,,,,

tulle que f

(20.1 [)

isr .g1 Sci.ent 1<p<q<Pi. Soit e(x) une fonction nesurable dSjInie sur f.
Ori suppose qua auELq(2) pour tout u E LP(i'd). Montrer que al5. 1<r(n) avec
Pq

si
F'

q si

(On pourra appliquew 1c thUrIme dux graphe ferm.1],

IV. 9

So it XcL 1 (9) un seas-espace vectoriEl ferr. On suppose que

11 Ll(n). 5d.

1) Montrer qur r1 existe p> L tel que x 4=LP(n),

ia
[Pour tout entier n> Loa pourra considher l'ensemble

xn .(fexnLle(liv) (n) ;
2) Montrer quin existe une constante C telle qu 1111 <CM: vfEx,

inilgaUt d Iterasen. On svppose que $Qic j uno fonetion convexe, j # +ffil.

Soit f E 1..1 (2) lel que f(x) E D(j) p.p. el


Montrer que

(!}E 1 5 M)

[71717" On 14',7..c.a.t. que vene contiw.1.1.

cor.vexes.

!ni

Soit. 1 < p <

Soit

""i

tru: fenction IrDn-

On eTirSirl.lf.e la fonction I :L111

--P ]-'",'I'l dfinie 131-r

j
J(u) a Eu(x))dx s 11:0 42 L1 (9) 19 4.. si j (u) g I) (51) ..

1)Monzrer que J est convexe. 2) nontrer que J egt s.c.i. [On pourra commmnwor par SuppOSIET que j O et utilser le lwr,ne de Fatou). * p 1-c.,~1 est dogne pir :L (0) 3) Prouver que la fonetion conjusule

3 3

J* (c(x))dx si
3* (f)

j (E)ELI(9) j (011T)(9).
et comencer par

s "

(Lorsque | < p<u2 on 1:urra introcH ira J (u) =J(u) n


dlterminer n

4) On d1;.signa par a3 (resp, 1.11) le sous-difIlrentiel de j (resp. J) voir problIne

So i trit u E L; 19

et f
fe .T(0

(r1) ; Diorcrer quc


Z(x) gi(u(m)) p.p. sor

E.gpacee L.1 (11) avec Sud.t 0<c, <1. Ori. pose Let (n) = {u ; u mesuraimIe et u:" Ll

1.to

u(J 11111)
Plua i.hrcillevent mont;cer gol (u+vj
.

1) 1.1 rifir que 1.1 (52) est un espace vectoriel r.ais 1 3 a n'est pas uue u ,,v Lel (2) u> O p.p. et v 7.4.0 p.p. mien!
Lul o +1,41

E
1 r
r
r

2) :'Iont7er que
tu.? + Vti,,v E La
.

C.

L&t

4011:

ce-In ve:f.e igEulr 1 unz:

th2dEd

C.

1) Simit i<p<7D.. Moritrer qu'U dft p) telle que 1.5-13 1 15

C(1.2 i P * ibi9 1-1 (1 4 1 11 * 1b1 1:1- 2 1 .

1f ab 1-1

ta dbrits que T.P(2) est i.milwrimiUnent cor.voze rniur l <'5-: 2. [tlenser a quesd.on. pr6cOder...i.e et l r i..11.11-,allt de ]1E31:1.er].

Ev.74] Sois 1<p <-.


Montier que p.Dur tQut E>0 erais te une const_Inte C >0 telle que

1a+briP-laIP-11.1Pi<ela1 15 +Cia rbIP YB,bEE.


1
1'

2)Sic (fm) unl luica a 1P(g) reno q 1) fjx)


if
LEI

t(x) p. p, II f p
Vn,

suite (f ), est borne dans m

Muntret qu.e fe L P C-1) et que lit I it n I P trsia 11


$-:

pulla appliquer La questi.c.r, 1) .sveu a =En troduire i peor 1>Ch fixa, ld. buite kpu I Cf ri IP -I

et b = f . Il est uti:10 e in+ P - I 111- - I trE-f

t
une suite de LPG.) 12t soit f f(m.) p.p. i..15 (5...) tela qL.

II
tforitrtv .gLie !E

f .
n p O.

s d'Egete. e t d

vi:

en s uppnse que
.11

< cc!

) mi.? quite d'e fc.1.1CtilDrIl CIC3vraSies tenis que E --h f p.p. (11<e. p,p.).

E) S2.1 4: e.> o 11.1se. (11.h I 1.E


11

~ j-

2) Plus pri 7 ei31-nent, isnit Sti (a) Montret que 15 (a)I TI 3) (Elrov). Mantrer que >

e,

>0 Phil <6 et


[Etant d r tel que

resutable tel que


f uniforiat Eur

1 entier, prouver Cn titilisant 25 qui I < et 2 eKiste CI3

existe

entier lel que Nik Nca , 't?. ;

if k (X) - EN) I <: a 4)

Nitati). Soit (En ) une suite de 1..P(5:15 avec 1 < p <. On 5UppOSe que

(1 ) o[ t_,7- .m:11e !es runiraticus LAI ames

[g

; 3(N) > el,

11) lillt>0

315:0

tel

que A

if 1 12*1 n

Vn

et

YA

mc -surable

avec

<3-

ii)

fn 4. f

p.p.

Montrer

que

f ieLP(9)

gt " qUe

f r

-4 '

daos

LP ( 2) ,

IV, l6

Soler $1 = )0 .

1) r-

On

COn5 idare la

surte

(f

de

fonctions

dlfnies

par

f n

re-"".

Montrer ciue

i) f

n
n)

O p.p.
est bornedarsL1 (51),

m)

f q. O dans 1.1 (U). n

iv)

t ty 96-

peur

la

topologie

of1 .1 ,1 . ).

Kontrer

qu' O.

n' existe aya

le sc.us-salte

ebaraite

(t

qui

ennverce

pour

E
La topDlo,hie fable c(1 1 .17).

2)

U1.

On

eonsidate

la

suite

(3n )

dt

fcaetit . nt

dZficiet .

par

lip

1b 151)

- nx e .

.r.l.' que Mor.h

i)

ln

P.P.

ii)

(sn) zst berrile dans L

Lii) gra
iv) gn

dans

LP (2),

11 ,LPI ). O pour la topolvgie 011.

Boit

<p <10. Set (fn) uni dans LP(n),

d2 1.9( I !)

telle

que

1)

(fn)

ect

borne

ii)

f -h n

p.p.

1)

Vontrer

que

f n

pour

la topolozie 1(1. 1),LP ).


quns

Rin pourra

eemneneer

par

lontrer

f n

f paul- 0(LP,IY") et

f n

24,

aLors

p.p.

(utilser

Vexercice ILI.4)).
1 'on

2) Mame

eonelusina

si

remplact
hl <=. et on

ii)

par

iiT )

fn-11

O .

3) On 5u pese

mootenant

que

fait les hypothltes i)

et ii).

1L
Montrer que HE q
4 0 pvur tour q, 1 q <p.

lon pourra introduire Les fonetiens tror.quOes T kf n (voir exercice 1V.2) ou bien utiliser Egorold.

Fonction5 de Rademacher, Soi_t I <p~ et sea f E 1.10e CF)


On supp-ose que f est T-priedique i.e.

f(21.4-11=f(E) p,p, On pose

T 7 1 1-1 f(t)dc.
0 On 'l'ansiare la suite (un) de TY(0,1) dlinie par u (x)=-f(nx), xe - ]0,][. n ]) Montrer que un

---f
L

dann 1.11(0,I) pour la topologia 0(1.P.LPI).


p

2) Dtermiber tia HU

c.

3) taam:rwr 1(J:: e;semples luivants

(;,1) .= sirL 4x, ui) u (y.) n


) o xE f (x) si et ,12[

xE]IJ 2,If. ladem=;ler.

1.415 fonctionn de l'exemple Ii) sorit les 49nets:ons

TI -571 1
1) Scnt {gi n) une suite de I.PM) avec 1 <p<ts et sois EELP (n).
pose que ; i) f ii) Uf U p f pour cr(LP,L17 ), + Wt1 . f fortement dans Lr sup-

Montrer que f n

2) ccristruire une suite (fn) de LI (0.1), 1:1 1 0 telle que i) fn f pour

ii) II 111 11

1 - In' . C.

ii i)

II E n-f El

3) D;n5:laer Cr aux rsultats de rr eizerdcl

On suppeSe que 111

:5olerte 1 ?

ee I <

Sit a :11 hl une fonction ccuelnibil telle qu2 la(t)I <C (1C1 7/9 + i) On ensidinre l'application A

VtEE
1.- ri. (51)

(n)

difirds par

(.40(x) a atu(x)),, Y:Grr,

1). Hornrer que A 12st continu d LP(:) lrt .dans C I (V~ ) Uvrt,
2} (ir, prend kci W - 10,1[. On suppate qua pt.ut tOUt GI F (U) Celle

que u

u rur ce-P.LP ) atoes Au

-- Au pvor rILIL I FY )

wrrer que la fr=rcird a est tr1=1

[01 1.0.1r.r util iser

'25 fDptious de Rad.1 , .neher

voir exiorrne IV .1.;]

2tar5L 1)
(1,1 11.

Erinctizr u
K:Ut U o e Leca) :Avec 1 r ,p

oft prps.c u (x)


! .:entter

E j 1w4 1' .
O danIT

Lui)reise
.

' al au $1115 sud2) Qn SuppoSE que U '1: L 0:1.) et 1 1,1e u (x) -4 O qlland 1%1 1 0. O vont

poor

6 -5 O 1 'E nsetie Cluo I > 6] e5 r. de mesure '!inie, W . Mane rar qUe 0 dans L (11.) puur (L , 4 ).
t011t

3) Qn peend

'1 0

7 ]U ' i covergil n U ns T i M) nk

MontTer

ji e q e.X.5 te aurune shas-suite (u )

pou y o. (1.1 ;L) ,

11 .M

1) Seit (fu) une suite de 1.1)(51) aren

et soir 1E E-13 (1:2) .


scnt lquivzlienter..

Nontrt; que les deuK prOpriltd5

1N4 ).

(A)

-4" :Jour la topoleigic Q(1-1..1... n.

Of a 1 (13) j et

e
el [E[<''''

f -4. f f VEt=1 :, E eicurable


E. n E.
et

2) 54 p= 1

1472[.(ix vUifier 14.e (l) ' -1 (B).

1) On Guppose nain....enant que p =1 et que


Censtruire

ini
V"

Montree que

(A.1

<B).

exente nersirsnL que, en 8,L;nral, (11)

kin

poi rta u1.1i9er l ' exetei ,te IV.21, qurstion 1)i,

1 1)

Soit

U) une suite da 1.1 0)

et snie fE 1. 1 (1-1) avec

On SUppOSe
j

a) E ZIO Yn b) ie ft

et f>0

p,p,

11.L

I f'

e)

f , n

f YE= E

E plsrAbte u et

Moutrec que fu f [Gr.


biourfa

dens 1..1 (1;)

per pe.7.uver que

F mesurab-Le et

V111.21 1 Soit
pY3r!

111 .11,

una

r. price ion orsut-21-..1 e e

1} < . hlr

pn.-

de mor L'r.2r que 11 ans ,...rb le

CD{oELP(i,72) 1 u>1 p.p.} e.st fartaC daos 1.P (n) polar La cDp:D1c.cl.t G(1..9,111 ).

rt

tensie .lee d r aboed le eas. L i

p<ts.>

) ?..:ontree que C

est

eonvexe et

dar-5 LP (It) fort.. En dil.duire .gue C

fern .,:eirr la topologic g(L12 ,1.Y 1 ).. Orb tansidre mainterlant le eas p =te ,.
2) ! .:entrar

1
ron

;1,P;"offE.P WeL1(r;), rzPeLie


f
EL

et tPLID

ter=eneer par supposer que

bens le c2.s alrtral on pourr..1

intrchavire L.> [Ifi <ft11. n.

3) Ea dduire. q 4)

e esc ftai dan p

pour la tapcilc.gie cF(1-7,I)). llontrz e que L'ensembl.e

Solenr f 2 E L.E. (0) avec f 1

{1-111 {F!

11

"1 2 P`P`}

st

coTcpac t

dans 1:(0) polar la tcipolnin

Ly suite

. Ei 1 So t u

R19) Spit

cl)

salte r5gulariserte,Soic ift ) une

03 N L (IR ) Lene que

II

<1

Yri et

f PP- sur E

On pose
1T

p n= n
V dans

( c n u)
m ) pvur la topologia o(1:".1.1 ),

1) gernt.cer que 51 2) Z:unLrer cLUC i

IV -Id -p O pOUr tOULC inule 11

a '

4.,
uhit

ri, zzN

iwt Illastcer q '1 i1 exis.te 4:112 SUitl) dn c7m) teLln riye u

1) SoiL

a)

111E n

13) la 3. 4l p..p. donP 45 c) u. u dans 1. (2) pwit la topolozin c(1.. 2) Si d.t plus uzw0 p. p. monrrer

1 ).

l'pn phaut prendre en cutre

L E

3)

uen9e dl.ns V6liEler gut e (P) eSt >

mur la co...ulnia

TIV.26 1 8tDit
mnnlrer que f

ti

un ouvert et soir: f E Li (r. loe el.: sel.klerit

1.1(l Si

Supl j1.5
et dans ce cas A = fir 11 1 .

c.c (0111E1)11. < 1 } <el

21 Hontrer que f+ E Lle-g) Si et selatement si

~~

B- Suptflq) ; tACc. eL das ce eas 1E1 Df +

IP-Pri '7 4

1/415:"" - D1 < 9'

.3) t'Ulises questions si. 4) En dduire que e f f19 ,"

'cin rer.plaee Cc (i1) par Cn1 (2). e

em e..0) \kPEC:(P)
7$

Et

(..ZT:3 O

111 2 22

j Soft nca

n claven. Seient 11,1.2e Ll (11) tela que u*O p.p. sur

un sous-enzemble de On supposh que

de mesure posir.ivet.

oe c:(11)

fuo> 0) . (Ivso o\

i"

Eontrnr qu iil existe une enrist&rit-: )4> O ten* quei


vi= .1u .

I 1;..571 503.E SciltZELI5 e.ii1.19 ) Mvutrer

LIZ")

que p

;,;
1 Ort rr) ql

P (x)

;(0x).

"" in *2

dArts L? &)

I.V.29 I Scvi,t Y, rIR,11 un ccuipact. Horttrftr que pour rout end.er n> l il existe une fonetlou rh telle

IT

a) 0<un Ir) u n = i

1, sur .1 sur K,

9, e) Supp un c K + b O , 1 Inillu rt( x) I 4Can i 'lil VicEZN va multi-entiar

(ca

e eh:. une constaott qui dpend !.:.eultrzent de 0.). pourra prendre

,:ible +15(0..t. [Suit xn la fo.r4tiort carectIrstlque. de l'ens2. 21% * Kia ).

izii.ent 1 .Qp
1

tele que

1 P q

vri pasa.

1 1 -1 de scitte que 1 .N: .Q.. P q

5.pient f E LP ar1,11 ) et g E Lel-ORN ). mor.trer que pour presque Laut


C5E intilareblt sur

E>ri.c don. y E-4, f(x-y).g(y)

tir

.
c1/p'

[011, pourra int.roduire a EI piq u i,

et .1crire

1f (xy)g ( y )
2) On. pose

- 1 f y)]11 Rcy)i113 ( 1 E( y ) I I- a g
U* 8)(z) = 133111,
(x`Y)1 (Y) d'Y.
r g, .

11 d)

Frani:Cr guC f * g l,r

et quef * 2, li

3) On suppost meintenenZ que 4 / I* P q


!-Soni_ret que U* ig,)C

1.
que Si 1 <p <" a1ors

c(0,1 ril..01 11 ) et

(e* I) (x)

1:1 quid [.r:

7 70...111 1 Son .1 - E 1.17.11)

Feut c!-Laque rO ort pos..e 1 f r Ix1 -1. 13 p Z) I j br,r)


711

1) Mantear que f Z LP Q1111 ) ti CaN ) et que f (x.) ---> r r 2) Mottrer que f

L.

E lona t<p

[Ort pi:1u rra .1crIr i r f L

woec cp r epmertablement cho -LsiJ..

1) Soient f .1E L1 (0.11 ) et h Vririer que I* 2) U1.1.1

1,0.) avec 1

m.

f et que U 91 S) Ir h= f* (8, h) pn supp.ose que Clinrallser au. .cas

1.2 01:14 ) ;

wN Go

a. )
EE L1 CRII) 142c

bki.ntrer que. f -

p..p . sur IR

rl

solt 1,EEL)

11)

une fanctioa

cpusiare 1'op4r.eiteur

MIS -I. L2 O?) clfimr, par T E,

r rY Kif i2r que T es t t'orna. DtErmialr T


A gutlle candition sur g a-t-o 7
T

* * TI et T T ; que remarque-t-oo

r
1 111.33 1 Oii /-xe Line f oncljo n 4ple C CR1) F
da forte tons O et on
o^.11dre 12 fa mi1la

oa

w(x4n) F ?t'EJ. .1) Sit 117.p


4C.n.

Montrer Tt VE >0

5 :15.131

rli que e7.


LP M1.11)..

T h

Vh

evec LhI<5 et ' conpuct

2) Murnit..:Er 1:0141:

n'est. p49

E'7, 5F
ezt

1..;o 1,t S'un nout.-enzmmbi de T-Q".111) p.var. I 'g. p <-.. On


k5.3 s LP CRI).

SUI2

verteri z.

blcintrEc que 57 as t bcyrn.G... Nntrer VE >0 96>C1 tel que

hif-1 1N <1 LP M 3) Mane rar que 5dc > Q

11111 aves

et VEer ique

11: puye et burrtl :=c).


N

1f I

<

V!'

LP CR VI)

Cenpar.er au corvliirt IV. 26.

ni. 3s

1.`.05:1

T PAhrm l, % une ECITLCtiDTL f 1}:19 avec 1

<0. et scit

oil 35 dZsigue un ticirml de 1.1 0115-

Itnattlr que ..7[ n est r.alat

eir.pact dari LP (2) pout tcLSL auve CL

111 . au C5re11.1fte 11l, 27 Inr.111 11.i . Compare:

J:?mi-intJgrobic. on dit qu'un spli-ensemb.le 7wL 1 (n) est triint42rabl.e s'a les propTiLltls uivallte.w
(a)

( 5-1) ( 1 ) Yest bGra.1 dans L1 W:>0 93>0 tel que E g. E meswabl.5 aves 11 mesurdbIe avec led:<:,-; tel qua f <E VI EY. Vf. e 7,

(b) ifi<e E
Vg>al a'AJC:n

(e)

soic

m y une
n

sulla erGissant d r tnSCUblaa msurbleS DV.EQ. i

I <-

vn,

cella

crut

I) Meintrey que :Fest


(d) lit sup

i et seulement si on a : 1 1 1"0

et (e) lim 11115 1([ IQ - 1.919


0,

-1) Ott relatZveL.Ank eobliact c!ons T)(9) alors7 2) Ilkuir.rer gut Gi 7=1.1 (

est zlquIJ.,71L4rale. La rIcti;tque ust-elln vrai?.2

( 1) On pe:Jc montrer que (A) est unt consquence de (b) et (e) si 14 mesure

est diffJ.se (ne. sans aLomES. ciDnE ij&er par "Empla rd =r1. 11 cu al de la mesure de Lebesgue.

23

CHAP I 'In IV

2) Nater que hn

ri.

+ f n + g). o

3) Nater que f ngn (En-f)gn f (gn-g) et que f(gn-g) converGene I dornin.e.

arls L P (12) par

1) Notar que 1.1 (1:) n1.(1) c:LP(n) et plus pr&ti_sifrerit


CQL11.2.9.

f Pf : 1 1

1.

n est o-f

1crit n.Uri ave Ir: :"<all Vn. U n nn -.7 1.1 (Z) n Cel) xT ffdr..ns n n-rl"

Etz.nt deinnil fe T"(n) alqrs x L(P.) (vut: exareice 2) Suit f II ql I r. q

ur.e suite .d.e LP (12) nt5e1:, teuE quAl.

dar 1.P (2)

C,ri,e.te & extr.: ire une soug -si.lt e cm peut wurpuner quo f. n

f pr (tbln-

raue 11/.9). en dcluLt du leame s Faitou que f T.1/42) et lif q <1.


3) On sait cuja. grce 1 la questian 2), que fET-.1 (51) et dore fELr t,.9) pour tour r compris entre 13 et q. D'autre purt on et l'on a r < Jf -f a f f nq < 11' p (2C)1-c. 1-a vec. O 4 i < I r p q

i) On a gfe If_|| ifp D'autre parta soit O <1e < I f


A=

.1-2 skr,] 11E11 < II E e. et done 1 2


p-FM

et snit if<x)I>k),

{xen

1A Alors 1AL*0 Par suite lim inf ilf > 1. Yk<


P ' 1 CP

et

11 e' lz_IA1 1113

Soit k>e et soit A dfini :orna ei-dessus, Alurs kP LA I et done IALI


C\P

"fp. IL en rsuIte que LAHO.

3) f(x)= 1131!xj.

r
1%1.8
On considre l'oprateur linaire T:LF(9) Lq(Q) d'fiJi par

L.

Tu, = Du. Le zrohe de T est farm1. En effet, soit (un) une suite de 'Y( ) telle

que u

dans LF(n) et au

! dans Lq(n)- Quite extraire me SOug-suite


f p .p . Done f =au r,p. i.e. f Tu.

on peut suppuser qua un


L.

u p.p. et aun

'Par suite T est un oprateur born (thorlme 11.7) et il existe une eonttante C telle que
II auli Vu E LP(ri)

ler ces 1 p<1 ', On dl:dut ae (1) que .

jial q hri < CgAvii piq Vvegig0). L'aplicatan y 1+,1, J1 41q1, dfnit une forme linaire LI.n.tinue sur I.Pk G et par constquent al e l. ( Pk) (5
2iele Inas
p

On choisit u 4 1 dans (I).

1) > t'un de la norma II II 1 est un espaee de Benauh. ?out ehaque n, Xn est un sets-ensemble feri de X (voir exereiee

X =X. rn effet, soit fEX ; l existe q> 1 tel que 'i 5 I + I in f e! Lt i (2) . Done f E L (n) dls que 1 +--.41q et de plus D'autre par' 1f II an , 'Ha., II
f II l II

f II

1-0 .n q

ave.:

In 1-1211 I . i 1 14, -Tn7 . . q

Par consquent fe): pour a assez grand. n

1.2IRCC a:.1

DlEkr, kl

exihku r tea. que _Luc

2b

11.

1- 4>

Suite

XeL

l+Un

().
I X esc un opa'alilur

2) L'identit

den.t

es.t

s t donc LrL oprateur born

Sc ir

On a
f(Y.)1,j(E(x)) + j

(t) p.p. sur

et en int'j-zrz;nr on obt.ent

Sup

fi) e j.? (L)}<AT


1 11 1

j'U),

e;11

111 1)

17SoiEnt1, u2 E Duy

et t E

[11,1].

Al..ctrs tul

Lb 47) est. nasuz - z.ble

(u,tr j .C9C anneinun>, j r autre yart

(tul + (I - t)u2) quc

Lj (1,q) * (71- t)jeu2). Enlin

ti

exis.iz des constzwes` ,2 ce b

j a5 +S Vverg,,

j (tu: + 41-Gu2) E L 1 (.".)

et

+ ( -1)111)

+ (1-0.2,11.1.2) . n

2)

SuppQ3ons crabnrd que j 0 SQ i 4 EV.,. Ii 1 ens2451e

L.;

{LIE

; J(u)

est fermZ. Ert effee son (14 ) n et j (i.L.a) X.

une ssjita de

(5:1)

telle

que u

.1.L dans

L,P(5:2) --I' u 1....p, .:; 2

t. . cin pflut S uppuser qui2 Qui t te . ext.:Eire une sous -suite

11 dduit du
Darte It tan
ussi J (u) =

le= de rz.lou qua j (u) E li t (5i) t qu.2 _:. f (u) 1_ Done J 'tse s ,e i i , ore pn52 j(s) = j(s)
+ afu +b[:17.1

-as - b

e, Alors

3 e.r.

t done

3) notons

d abord que
Ir 1/21 ir

(r)

1Z)

(f)E L I (P).

elEet

on a

fu - (u) < jir (1) p.p

et done 'y r f u - j ] ( u )}9: 13* ( f), Sup{ u ] L'ingalit inverse st plus dlicate. Qn coutence ptir supposer c.ut I <p <ma.
On pose

r
On va .montzer que

4(0 = (t)

tj15 ,, ter,.

Erg

Boit f L91 (9) ; pOur presque- 1:out 3cE Sup{ (x.)n - j uerl. 1 p -IuF }

fi-N

ES E

aireint en ur, po

,atique not u(x) tel que f(x)u(z) j(0).

11 en rLsi 1 te quo UE
r *

(-15 (r)

et que I

F- 1.- 11:12) (Pobrq-Jci nver,le, (1) .

tune j (t):::1' j (f) ; 1.1r on n n rc urnz J.:: 211.1. , en a .5 < .T


Ir ft

i....e,
* Iii f i ri ery Ir:" i (1)

r
1

DiaurtP pi': ` t., pour i:out ser", j ri (s) : j* 4,S) eltari (voir elltreice On ocndur s gr'iut

u e - en 1.1.7 1.: 41. :1 '4 .-i i 1 VI' 22) .1t or p 4c r1e ensuite corme l'exer.t.irls 1.23.
ru 10

,s.

,g-

ti

_\ 1 '.

converzer_ce rrxmcrione que


er

i f

7.1(1 ) alors

ii j (f)qL1 (2)
I?aris Le cas
r =

(n

on reprzr4 la cnIthode ci-dwasus, par LIcemple. avec (t )

4) Cra su pose d'abord

f(x) e 3 J., (11(x)) p.p. sur P. Alors

(v.) -i(.()).,,,,,,(v -u(x)) VvEt et p.p. saz P.


Pranant v=0 on voit cluC 3(u) e L: (1.1) et done J (u) j (v-u) Ifve D(). Inversement supposons que felli(u) alors on a -TM +.1*(04E15.u.

Ponc j(u) E L 1 (n) j* < Circe j (u) +

o E L1 (9)

et

{j(u) +j 'Ir () - fu} =0-

n
Li

(E) - fu >0 p.p, on conclu que j (u) + j(f) - fu p.p. i.e. f I Di (u) p.p. u

I IV 12j Ori pene

EJ

f v'

et p= I

. u

11 s'alfa d montrer que

(!)
On DGae a ft ct

(If Y1 + (f )
g

<

ilk f

ipp l/ Ply u

b= ) L de aorte que

a P bP .1j-1 f bP- 1 8 < f(yobP) LIP I (flinP) I IP


a P4bP) fp ( 4(EPi-zP)1/P i.e, (l). D l oCh (ar-pbP)11'411 j(pisp)1 /p .

3
i
t + 1 - 9111-111P)24

1) On 5e razne a .prouar que

Inf

t N IP+1 ) 1-5

- 2

tE

1+ I

1 t-i IP

fi

1.1 aulfit de wIntler que

{i lel re[-1,91]
pc la fonotion Lp(t). 1 ti ' L 4.p(t)>0
1 '"

t IP + 1 - 214.P
I t-11 2 O.

211 11 9 vrifie j

Yte[-l p +13, 1,p(1)i.lip1 (1)-0 et

.4"(I)>0.

1) On se rarkne montrer que slft>0 Ue >0 tel que


.-11<ELti+e 1lef1I P -it1 15 c VtER ;

1 1

ceci .1quivaut

i (+II' -itiP- i
Itip On a Co<lin vergence dooine que ee lf1 1) et

o.

akitre pare (Pri. 1. D p.p. On en dIduit Par con-

O. Par ailleurs on a IfI P- If n-f1 P 1

et done r iI1fniP it vent litri aun n441 et par consquent ti lin 111f I P n rrfro, 3) App1iqurr la question 2). -fl P i =O. Ve :0 If :1 + 1( 710P if n-f1 P 1 <141

D: I 7111
1) t 2) Soit gn i.rxs et tZ L ut 11e S t.), Almrs (fur.,:tion elraetflristir 11 on en dr.:12,,,iit par corf-grsenee ...1"9") que ign ic` is n 1 1- G. 3) Soit m> 1 ntier ; on appliskda la ques.dor. trouvet un entier N tel que I5NII) al Ort pose E I S, m et on a done YKE Montrars quli E n f uniform:: n: 2.21 :1... ee 3, = On rwat. de =,

f k (1) -E(x)( qQ-1- Yk> hl


t

Ork pose enf in A U E de sorte que LAI =

sur ri \ A. Et3nt clonn >0 on fixe un enti.er ae >

On a done

I f (x)--f(x)1 <e Vkii- m 1 V>: e 91 E rao k et a fortiori 1 f (x)-f (x), <e 'VI> m k xe In A. i) et puis Ac gr;1ce

4) Etant donal c>0 nn fixe d'abord 6>C1 grhe

la question 3), On a done

1 et n

I if 1P"e Vn A n + f uniformmane sur n\A. . < e et rf IP

On en dlluit (par Fatou) q ue

1A
9 1A
3e+ Ifti-f1P2/ L 21 \1 j

lErt-i1 15 + A

FIV:al l) Notar que fria --. 0 kikpeCa(n). Supposons, par l'absurda, q ue r 1*). --`f pour o(L1,1. k

Alors ffimg = 0 \-kpEC(11) et par sume (lema IV.2) f =0 p.p. D'autrT pa:t, on a jfnk-4. jf I 0 ; mais 2) iv) Voter quo f. 1.D den-5e flans LP 1 0) (ear p>1 et -t dt --* 1 absurde.

0 WEIC (n) et utliser te feil. q ue Ce(Q) elt c

J
r

1751,! 1 ._17 1 . 1) Prouvons d'ob3rd quQ si une suite, (f) f b) Ir a alors f =" 1. En aftet, ih existe uta suite (gn) de LPU) telle que c) grL E convt f n, f d) gri t 445215

f pina. la tppolnie 0(1.1,LP'),

u
1!

p.p.

tortement

I F M.

Il 1 . suite de b) et c) que ga D'autra pare (thr5orlme

f p4p. exste une sous-suite (s ) tecle que nk

0.
f p.p. Done 8k f p.p.

1, pour

Montrons maincenanz vous les bypothlses i) et ii) que f topolosie On Extrat une sous-suite (f ) telle q u.e E nk
P

f pour la topolegie

u(Lr,Lr ). GrUe a ce qui prcUle on a f .7. p.p. L'unicit d La limite" en-

et Une
lis sklp fif T1-fi q (20 q 81-(.1111) VS .5
11-141

1 1V . ,3 ]
(1i-a)7. On en dduit que u -1) On v rifie ai:55;nent que j u (t)dt n Li (ear les fenctions et escalier sent alors denses dans ) si I <p E Lid(E1), T-priodique,

pour

LiPt ). Lorsque !El) (11) un approrhe f par une fenction ice tiene qu if-.g; <c.
P

Soit v(x)

g(m), xE 10, I[ ct soit LDEL I jun.11) - 4,1 2.1 II K.P II -

On

Ve > O Par r_Qr.squent un 1 et done 1.122 lu an 'Ju r? -1101 < 2c II c7g 11~ topplOsit j!ip r, rT = 2) 11a Eu ir.. n p Ti 3) i) un
.E)

pour la

O pour la toriJ]cgie o fL7 ,L1 ) 210.+ pour 14 ropcitggi (t. F T. )

tin

2) rart. On peuc 1) leit (un) unC suite de LP (51) talle que un -- u dann LP(1.

extraire 1.1n 5011S-su i te talle que unil(x) VE . 1,01) (thorlme .111.9),

u(x) p.p. rt lu 1 <v

Vk avec

On en ddul.t par converger--ea domina que Au

.) Au dar_a Lq(5n P- Au

V"unicit de la limita"" perrnet de conclure qug lotete 12. suite Au (d ta 1.1 ler ! )

2) On utilice la suite (u ? die Vezercice 1V. 15, question 3 ii). Alors


au r
a

e)). On en dduir que 2(0413) et Au -. , -ta(a)+a( n ki 2

(0. 4

I'la(a) 4 a ( ) ) Yei , e

t done La

fonction a est ncessairement uffnr.

iraine que erute la suite (f ) conver ge faibleenent vez-5 f (dtailler ie raiu sontement), 2) Cn extreit une aous-suite
qui

nk

) talle que E

-4` f ptp, D'apr's ce

prcIde on sait que f

f pour la

topologle a(LP,LP ). runicit de la

limite" entrame que tem te la suite (f u) converge faiblerent vers t (dtailter raisonnetent).
3) l'are

mIthode, On crit II frif q < -f + t II + a q rkf En Tkf n q Tk f n Tkf q

(1)

PeUr t, .t k >0 on. a [ f fn141 . f i f n-Tkf ti lti< 1 ifj>k] a


p l utre plrt

oa

2 rI

1Pcri
111

er donc kPqf

If Il<CP.
lifrjkl.

On er (2)
L2 1 per,Q 41

c:ue E f -T n 14-f rAtc. ,2. enteiatr,e qu.r. II f-Til f II q cP \l /q q Va,

( GroYIS

(3)

CP ale Etairt donn5 1:50, on fino k as set goa d pinur que (-21 <t, Par conver. kl-li tel que O et done i.1 Exsto. dominie cn cit qu2 II Yk f A-T k f i1q r1-0,

(4)

E Tkn-Tk f II q < g
%D.n

sdn, N.

Ca=binant (1) (2) (3) et (4)

obtient

f ri-1 II q <3f, Va> N, nue 2, ...Lhode. On applique Egorov


tel que

tant dann 6>0 YL existe Aen


crit

!Ai

et. E

f uni.forOnieut sur in A A f n-f


4 I fn-f I

lf

el.,(2isa

+ II f -f li IA11-(q/P) p n (2C)11

Ini q. L (0 A.)

61-111/P)

v. 271
I) VErifier que f u (t)dt P- O VI intervlle borr. Conelure E,r:ice 1 la

den.sit. dans 4' OR) des fonerions en esealier, 2) Voter que I un(t)ds .0 VI Intervalle born.. En effet, Gtant donn E>O, on fixe 3>0 r21 que 1(11%1 11 . 1L1) u (t )dt n Qix chois i t 0n a abra ti .(c)dt ci 14.n

<e.

en pose

[iu0 1 > 6], Chrl

u .1 u III-T011 e ICE4-il)n.

dc. serte que I (I-in)

Er .5 \In> Y Cpourquoi est-ce possibe

ilin(t)dt : j c,nalure grIce

ilL <:e Va>

la densit c:ans LlaR) des funr.tiens en scaiier. 4- u dans L1 C1) p.= Q (L1 4 4.4.) k
OR cons'id.brc la fonc-

SuppoBens que urt tiesa par

Alurs ju n k_

(-I.

ne converle pas

1 ry.2.2]
prouver que (B) Pi (A) utilser le fait que lias:uce vectori el an3andr.'3 p.nr lee fon(tons 7C E avec E nesurable nu 1I1 <os est dense dial Lr (tir 1 <p' <ed) .

2) Utliser le fait que l'epece vectorial

[hal . les fe-rictious

Ect9, E m2surable5 esZ dense dar L21( ) ( justifier eeLte assertiun 1).
ti) Etapa dor,ft E>0 Cirt riNe wc t .13 ner5 .11r2bit 1411; < et

(1)
CID a

l<1

(2)

f .,(j f

,Fko
et' donc

fi .L)+( 1

,)
2

I E= j.

E 4 , o(rJ gtrula d b) et a)

DI autrC pAtt, .n.11 a

r n cc dpne

u=

17

I -

o(I)

Intu
(1 . -C)+D(I).

fF Colabinant (1), {2) at

JF

fI

rn.

11

(3)

ou voiL rc.us
+D(1).

lirfa-f
Oh, en d.Eduit

F Lt -1 utills.li Le P, F
I.L

-D .

j f.

pJC

;y2n, J7.- = +,!ri i

p?ir Len

fencticns

c...P.2c

Ir I

e5

der.z... danl L (6) (jut::_fier

FTV7571
1.) 1 -1.7. 2.! ) ni_[ te

de
)

C L.2 1 . 1.e.

11. 11

tito

de t,3

fw.11;2 p-p

On paur

Trnill-nwitE . (L
2)

r,ue
iti

-4 u p 1 ii

so:l.t LIE 1.72 () 1-.....1 rIv e

r .1
1.-z1 L qdr u> f
P'

n fi [lEi<n] Tk Suit A ir [11 < ) isane

sir; ti
quin

L; 9.
+0 et rione I E -u! 1

n
1.15. r1 ZI P Slzr cc.p56'quent 1A1

0(i LL

3) 1Zoler que pu:- LPE

Avm

14 1 12) 1. 1

e,!nb e

1, (u e 1.111( ' I

j141,0"5 .C1,411

't

e LEA

pour 1c t:...pribn,ie o (T.7,:r -1 ) .

:2.14 I
11) .Selit :reL 10:::r)
11' K.Pd ! u ia

(;)

jui: (15 n

.1

er done

p149.91 s

2.uH

VIn

+ RuE

,01 . n-0

Le premier terne tcrA ve=s O grace 19u 41110~ IV.22 el le st.eond terne tend vera O par eonversence deminc. 2) Sepit P050115 y n s, fi(x o ,R) cc snit 'g. la fonerion rarictristiqufl de 1(1<0,71.+L). alors n v

=p or xu) ft a

sur B(x0 ,B.). En effet

SUPP(VII-vn'ic:Z(0,D A-S(X0 ,?..+]) . D'IutrE pare ivri-v I

IP T1*(1; 3,1-i)(11 1

I (15
1r4
n

L.

.111.

.1 F

r
1: Le prol.onement de u per 0 en deheirs de .71. Soit {xe
n

Li51:1 X,n)

41t

<
n

Spit
i fi

gil

(resp. 0 la fonction caraccUistiwa de

(resp, 0) de corle que

C lur S11 . ot pose

v = p (C inik a
Alors y n (f:) et grce l'eilecice TV, 24. Qn a
O pour tete boule 3.
1.

Pour duque beule

peut done extraire une 5c.u5-su ir2

qui converge vers

u p.p. sur B. Pul- un proellid de $uite d: gomal or< construiC une coas-suita
ez.c.traite (.J ) qvii converge vers nk

p . p. sur

,N

1) Supposors que .A.<*. Meintrons que f 1.1 (2) et


I ftpi A ll LP:I n V..pe Ce (9 ) .

1 <A. On a

e
$oit Xcri compact et sait y C Ce(2) aves 0;:.n Snit
et =

1 sur K.

; on corta idee une Suite lun. ) de Cc 42) teiie que


E ti II iC I; u II Pa

et

"I

p. p

sin. a

(voir exerziee IV.25) en a iffiPu n1 A la limite bar convervnce domint) on obtient

If

f1/11 1

Allull. \fue T..(r2) .

Prerant u - sisrm(f) on voit que de 11 on g d4duit que fL1 (n) t


2) Srppzgons que B

'fi A.Ceraine K est un compc.. .rt arbitraire

On a t -4941B
f

VOE C (I1) tp. c

Reprenant

a mlr't mlithud.a qu'al la questiun 1) qn obtient

Niu E 1,19 (n) , u> O. Prenant u m.X [f>ji on volt que f+ CF., EC.

.
et sciertr,

enEmere par examner une situatior. RbstraiLe. Soit E 1,;.:1 . doint formes linaires sur E tt.1.1.es que f 4 O. On suppose q! (ct)E E et f (49) >0) (z.(q)) >0).

On v Mentrer
fiXe IP

exil.te une r_orstants I>C5 te11,e que gil. hf. Pour cha u2 01-: E on

E t tal que f(141 ) -

f(49- f(I,P)(9 + E1,0 )=E > o


0

pr

done *Fi f (;P)LP0 stP O Ye>0,

'ar suite .00 00g (0 1, )


Posan 1,P

VIDE E.

O on obtient

E-PC:(11), Ey = fulp et

0151 1,

limJ o
(1-0)r= q.

1) et 2). Notc.r que -I p

= l

et d'eutre phrt (1-6)r p,

On crit (x tent fix) f(x-y)0(y) =kZli (y)1112 (y)LP 3 (y),, avec


Dozw

1 (y) = I f :.x-y)

4p2 (y)

j g( y) 1 13 et 03 (y) - L f (x-y) r

c(y) I I

' a' N [pi E L 01 ) at 102 E= 1,9 ter,) .


Par eilIcurs

ItP3(y)

l r - 1 f(xy)11%.(y)1q.

Grice &u thorcle /V.15 on sait que pour presque tout xERST la fonctien hp3(y) i r est intgrble. Apr;iguent 1161der ('oir exercide I'1.5) on en cIde. lt que pour preagut t ,.I.ut 1.=h. la fnction y 4-* f(x-y)g(y) est int4.rab1e et lir fir( -Y)i l g())!IIV HEU: 1 2. 1 (j[E(21-Y)[ P ig(Y)1 11 Jy)

1(hz)001r <
GC

ofil l tr

flf uy)111i.1(y)1.d y

uar c...-164Jeat 11.1(9.g)(E)[rdxI.

ar lIgl Ir lif11 7 0stl q = eff r EeP T q P P . P u. q . L iti-

les li U

3) Si I, .1-1:1< ,d. e.... 1 <41<id, ii existe des suite& i:f ) et Iz ) darbs C n e n n ) et g que f Lel a ). Alorm f ilbgriGcc ) et g da dares LP n n n e 5 - (Vis) II -, 0. : n Par ensquent (1q)(50
O gumnd lx1

EI4 .3 4

. 0 on recouwre ,r parun nombre fini de boul ,ls Etant donnl 3

11(f it e). 2) Pour cbaklue I, il existe d >0 re1 gut i ll-thfi.- `i II (Noir le=e In: <e TheP.11 avec ihi <d i Lv )

On

1:10 5 e 45 =

Hin

V.Er if ier que 1T f -ni


<3E lilt1.11

I i..11-11 aves I-`1q< et YfE:F, que

3) Pour

eNl te fi- cP11 ouverl born <E,

I E i pN L huar an pcsc
p.

i)

Vriltler que
L P CR. U)

<

E ;7,

CHAPITRE V

LES ESPACES DE HiLBERT

Dans toa& la etete H dcigne un eopace de Hilbert muni du produit sca-

(,) et do la norme aszocide 1 1.

17 .11 11

Identit

paralll-ogYzerte
paralllegrecoe,

iL E un e.v.n. tient la norme II tt vrifie

alr b r 2 +

lia,..b1 2 ..2(11aW 2 +11111! 2 ),

On 5e prot-ise de montrer que l'expresslon

(utv) z
Jiinit

2- 1 u

2)

u.0 =7 E

produit scalaire te1 que (14.1)

1) VIrificr que (u,v) IL(v,u), (..-uv) 2) Montrar que


(u+v

- (upv) et (ii 0 2v)

r 21ful i V)

4/L1,V

E.

(u 1.51) + (y r w. ) vt! v

E. avitej(i)

[On peurra ar'r a.u, my > GO a Ru-mq,


b= et

(in)

IP u +;?-...id

31' nonti:el- que Dm o v) a 4u ,v)

EIR

Yu,v r: E, A CM

leonsUirer d'abord le ces 15,:i >C1.9, puis A r: et .erti Conclure.

soit 1 un eepaCe asead muni d' une newure existe A;'11 meSUrable aves OZIAI

On euppose quril

On =adlre l'e5paca tP(n) avec 1 4 :134.fa muni de sa ,orne usuella 1 1 .


1.0ntrer que R g na vrifia paf l'i1entitQ du paralillogranne, sauf

si
(Don pourra considret deu lonceions a 5uppotts disjoints].

iy,31 Sqi.l.nt (u ) une suiza de H et (t ) une suite de 1Q1 n

1 1.

que

(t -t u u -u )AZO nn m mi n m

Vm,n.

1)On 5uppose qu la suite (t ) e5t cruisBante (non ncessalwemene n bprtve). Montrer que la utii,te (u ) .converge. n

<

peurra ommenaer par vlrifier qu la cuita lu 1 est dbroissantel, n 2) On cipmlost que la suite (t 1) est dcroiaaante.
10fttrer Libe ' i on a Valternative

ou bien [u n

-4'

(2Y) .511 1-len (o r ) ennverge.

si c -- 4>Q

airare (u ) converge et si e n n

lez cas

et

(Li) peuvlInl; s'E produire.

j ,bolt Lcin un convexa; fenal' non vide. Stie.tt E1FH et m ontcer que
t._.

f.

iv-u12 iv-f12 tri d,Sduiyo que


kr-14

1V-11

11 *

InterprItation OorLatrique.

u
grit~9T"P"IliTurim, rtir7
EP %-

1) Soit (KII) une suite doroiwanto de convexes ferrus de H selle que


I

n liontrer qu4t, ?our triut fl:fl, La suite u n .i?f converge (ftirtement)

vera une Limite ; identifier netre lmite.

2) soit (K ) une suite crirsanng de eorivexes Eerns nchr Vides de R. n Mmtrer que' pour tout f . ge (fnrtement) vera H, 10 suite un PI E cenve.t

L I

une limite ; Ldentifier tette limite.

Sois 13 H >11, un Ennetion coranue et !aoville infZrieurement.


MJES re r que la suce Kri converge et identiEler 1a Unita.

1',IJ Propotion =dilate lur la boldo


Schit E un e,v.n. dt arme li .1 1 . On pose
u Tu

unit.

si 11 u II C 1
U II 1.

Moatrer qua IlTu-Tv H 2 2 u-I. 1


2) Muntver que, tn

u ,I

E+

un Oe p,lut pis amM.inrst ta constante 1u21, avec (Lel ,u2.)),

. 2, L- ngidlrer 2L muni da ta norme II u II = 11111

.l. un preduit scdluire 3) (lue pue-ore di re si la norme q II est ssAvii-

nr-7uit

jfl $142, un 12311e conveui

Kett un ceine nonvexe d2 sommet O, c' ea t-.1-dice jv e K I I 1.1 '71.1 ,


E

> 0

* V

on S u p puiS e de plus que K gpt fEE H

t ferftl.

morare" que u =y lt

can feriad

por Lea proprlt1,5!

(f-12,1,1) - O et (1-u,v)40

""-- "-"m.~1w-2-~ u-N-quoAl-4n

4.19.

soit
h

un. e4paca neSut

mun d ' un

mEs..:re c-Einie.

[Q,,i-us urbe feinttiun mesurahlt.

x=lue 1.2 (n) ; I u(x) I <l'O) p.p.

SuT

lr' aririer que 'A ase un cenvexe i erm ruDn v'..de de E = L 20) Dterrniner f pvUr eab.J t f 1{.

19,

Saiient A fl et fi rI deux corbvexes ferris non vides W.5 que b est borra. On pose

n =e

at

1) Montrer que C est un. cacivIke 2) Oft p 5 e'+ P O et on Icrit u e


0.

-b avec a e A. et b E B (deci est ~~ ~~ o

poissible cat uE C)
VDriEj. .nr quPle -b 4 1 c fAl.erT.Jirber P b
3) :1 131,..1 . al C

dist(A,B) w Ir--f
aeA

bB

et Pp,a
El ust

h et h l

asure coupie rzel qua 1 2.1-'5 11

dZat ( Alln.

11.antr2r que Consi7rulro, 'un ex.emple aa 1.11 .cuple ia ,b0 1 eit unique (resp. nr mst p.as

an111.19.1.

deffl dessins 4.) Donner una d'uli.stratian fume tse trique ' dans le
du

thr.r:.7e da lann-Sanach, deuxar.e

CAS ' un espaee

de dirSert-

L
sed.r
Sois t411, 1 -p E urce fanction cciriveze de darse el. carivexa et sd. t Montrai que les pwcpritias suivantes sont equivalentes (i) <P(v)

vveI,
>0 WE X,
v-e 1 2 avec

(ii) r.r (u)


.8mempla

Iv.
Seis

Soit 1.11(=EL un sois-espace vectoriel firm' non rduit (0).

hl

1) Montrer que ~~ = Inf (f,u) 1.11M


1 11 1'1 ebt atteinten un point unique.

2 ) Spient UPL ,

ti> 2 El et soit E l' espace veetckriel engendr par

NI',

rP3i. Dterlmn2r n dona les ces suivants

(i) H - E.
0.0
Ci t.&

M I El .
Ixenlner le eas particulier&H ... L2(0,1), WI(t) t, W2( )

__-----

193(e+= t3 ,

71717-1

computJan d'un espace prdhilbertien,

Soit E un espace vectoriel no.rme xoc3Pe inl = (u,u)1

du produit scalaire (uiv). E muni de la . o.mplel: t^a 4it alcas que E est pac 1(.1pp-us ,t

j
prhilLertien), Variner que le dual E'. muni de la norme duale 1f

r , est complet.

Soit u:E

d
ci

l'applcstioo dlrinie par <c(u),v (Li l y) Vu,vtE!. 111211u rundition i cut-elle

,E

Vrifier que r 2t une isgmtrie linairt ; surjeclive 7

Ore We propcse de moutrer gue R(c) est dense dan; E' et qud E' est un

u fl rjj
11

espace de Hilbert pour la norml U H E , I) Transporter sur R(0) le preduit scaleire de E et te praleinler R(c)
en un prodvlt scelaire not - f ,J1)), f' BE k(cr)

Vriner que la norme asocie1:1:f,f - eonuide.si


MOntrir tiLIC l'en a - <f1 y> VVI:E, '1FERIO)

rb CA-si/A.~4 %-LS e CIA- tAZle

&S u

vtrctr

AJA

[ftnnt dunn fEEE' ore peurra rransperter f .En une forme llnaire sur g.(13) at
appliquer ensuite le thaor'Ime de reprsentarion de Riesz-rrClehat dana R(5)1. En dduire que E' est un capare de Hilberr pour la norme I n 3) Conclure que le complt de peut 2tre identif EI .

t'.

vim

soir E un e.v.n. reun de la norme 11 H E. On rappelle que l'ap-

plcarion Je dualit F est dfinie pour claque uEEE par

F(n) u {fEE' ; II Ell e.et I) On suppose que 1 Wrifie la proprit F(u) +F(v) =1' (u+v) Montrer que le norme Vu,v e E.

est associe un produit sealaire.

(Yin pourra utiliser l'exurcite V.I).


.1)IrroenlemnrsilsrlormieHH.est associe un produit acalaire, que petlt-en de F

ter. .7:zurra avpliqucr des rsultats des extreiees V.I2 at X,1].

r7 "-7 1

Snit a(u,v5.1211. --+P une forme bilinilaire continue telIe a(v,v) o v H.

Mentrei7 qu2 la fonetien vEH 1-* F(v) et dterminer aa diffarentielle.

gi(lopv) est convexed

r.lasse C 1

p77751 Soit

cc=11 itin s.Jus-espace voctoriel dm. 1 P es?ace de Hilberl.

H.

;:oit F un esFaca d Banach. hit rt :G

F un nprateur lnaire cntinu. 7 tel que

Zar

Montrer que l'on peut prolonger T en un oprateur S :Ii 151 DT! (G,F)'

x(9,r)

1-1

-"" '"~ P`. ~~~,~~~. Z#11~ ,91 I71S


r

v,

fue

v.15

Le trplet Vc:Hc;V I .

.Sait K un espace de Hilbert muni du produit senlaire (,) et de La norme Assoce I I. Qn identifie H et son dual 11'.
Soit V CH

sous espace vectoriel, dense das R. On suppose que V est

muni d' une norte U R qui en fait un esraee do Banach rfliholf. On suppose continue, c'est-a-dice, onfin que ll njection ilanonique VetH est --,_,--1,..X.4.4...

Z5v l r-47.frt -2 L --------1 en considlre Voprateur linsira T : H I Vi rlfini par

1) VIrintr que ITfil v ,NZCif;


2)Hontrer que T est injective.

VUz - H.

3) Clantrar que T(H) est dense dans


9 u,it E V' ; montrer que (PE R(T) si et sculement s' i1 wziste une + alvl constante an telle que Kcp,v> 1 4 VI.. V.

Soient Mi lic:H deux sOUs-espaces veelJoriels ferms. On supr,nse


Tie. 11 ,1 tir.fitTC; (u,v) 'm O Yu Hp Illtv E N

qua 1-1+ N est ferm.

Soient E un Baruch, H un ailbert et TEL(F,a). Montrnr que les uivalente5 ptrrlriztls suivante5 5:int e l
(i)T edmet un iniptrse gauche, vu e E

(ii)il existe urce constnnte C telie que Ha ICITul

50it (u ) une suite talle que u


d

a Zaiblewmt. ee SuppOile

iwg 1iCil

S 119

1 111:11

Montrar que u

u foztement, sane faire apFel X la proposition IIT.10.

ELUI Soit S E.t(H) un opGrateur tel


i:Qintrer que

quC (S40 O VuC H.

fi

V.rifier que I+ esz bijeccif peer CaC t>0.

1
r.

Montrer que!. 1i (.1-t5) -I ff, Vf E H.

4-1-=
[teux Trtthodes sortt pass:hiles a) Envisa8er Les cas 5:(S) et R(S).

b) COmmucer par tablir la convergence faible].

P721 ItrJes de contraenano lindaires et le thorlme ergedigun da


LO Kalcutani-Yosida.

r
SIDit TEE(1,H) avee JTfi ' I. Soit

On dIfinit o pour tout entier

n et

1.1) =

un(E) -(" k 2 TrE.


On se propone de muntrer que. lir 1) Vr if 'Lex que n (t) =1.(n 1.1 (f) n
nit

:14,1-T)

f.

sTst t ;

bloul_rer qu'U existe une constante C tell.e que

:?) En dduire que, pour tout f eR, on a

r.

lira

(f)

=1D N(I-T)

L
L
et

) On pose i iat(IY1). Mantrer que

u-suj 2 + Isul 2 < 1(11 2


Ea dduire que
! s t u..s1. 4- 1 (1 12. < l r u i2

Vu E H.

Vu E H.

i=0 tu l I Sri(u-Su) I

Vu E H,

5) Soit f E R(I-T), tIontrer qu'U existe une constante C telle que

;
n

-knrwir9614Pariwnzpurrz~~19~~,~p~~r~a-nr~~iorzirii

.~..mr-r....~

6) En dIduire que,

pout tout
1 irn
u n

fH, on a

(f) P

Soit C'H un
c ' est-a -aire

zonve*e ferm et soir T :C

H une contrartiOn.

Vu,ve C. 1)$071t (u ) une suite n 113 74.2ntrer n


de C telle

que u -Tus -+ f Certement.

11

u faiblement et

qua u-Tu = f .

Crnmencer par le cas 0.:j e =EE et utiliser 1 " ingzi1it ((u-Tu) - Ckr-Tv),.uv)>0

Vu,vEd.

J
En , !.duire que si. C est borr et V,e)=C alors T adteet point fixe. l -e)Tu ea avec a e C fix et e >0, -+ 01.

2)

1Considrer T u t

Ole in4gaIt dy 2arantbs/74... T 4:1->C1 et


iu l

H une carwraction. Soitat a l ce ,.. 1. Soietit u 1 , u t

des rE.al.s 1,2Is que

- ' u a des lments de H.

OF p-C11 C

4ue

2 m iui. i.1
,
a_Tul 3. 012 .
L

ITo -

1u, -u

1 2 -ITu.-Tu.121
4

1 7.1"1

(Cm polirra crire

7-..,

n 2 p Hr-lcuvul-1,(La.(ro-Tumr-Tu.) 1 i., 1 k . 1 3 pis' J


et noter que (3,b) =1(181 24 1b1 2- la -bi 2 )].

]
:1

Qua peut-on

en dduire si

est une isomtrie ? (c'esr-I-c;re I TLI-Tvl =

1 u-va Vu,v E
Ey7771
e

El).

ProprUt

de 8anach-Sbro7

1) seli_t ttii ) une ?uitgt de H tette que u


n
Lb

O raiblemant, Construire par


'kr ct u1

rcurrence une suite extraite (u

) titile que ni

-i 11,4 1

n.

converge fortement vers O Eu dduire que la suite (o ) Ufinie par o =u ? p j=1 nj

quand p (On poutta estmer le 12]


1 1.

2) Soft (U) une suite bornle de H. Montrer qu'il existe une suite axtraite (uni) telle que la suite (o p )
converge fortement vera une limite quand p - *. dlfinie par d ni PP ccmparer au Tsultat de l'exercice 111.4.

fi
r+
[V77]

Variations sur le lemme d'Opial.

Roit Kat/ un convexe ferm non vide, Sait (un) une suite de P. telle que, pur

choque vell

la suite (tu -vi) est dcroissante. n

1)V6rifier que la suite (dist(u ,K)) est dcroissante.

r
r-

2) MGricrer que la suite (PK u n ) converge fortemant vers une limite nor.e
e

Ey.

iOn pourrd utiliser l'nxercice V.41.


3) IJ

suppose le qua la suite (u ) vrifie la propeit


n

01

rJur toste sz,usi-suite (u ) qui converge faiblement vers. un nk


,..:;712nt

u I= N. alors

MOntrer que un --L L failLement. 4) Oo suppose ici que Lj (K-K) .11. >>0 nontrer existe un ulH tel que u
6 11. 5) On suppose ici que Int K9

u faiblemcnt et que P

K1

L.

Montrer qu'il existe un uE H t'el que u -4 u frtecient. ion poutra se ramener au cas 00 K est une boulel. ( u fu 6) On pose o n 1 2 vrfie La propriSt (P).
Montrer que e n n

) et on suppose que la suite (o ) vifrifie n

faiblement,

ira:

(en) uriC base Eli lbernenne de Lb.


J

1) VC:rifier qua e

ri TI-hm

O fai'blement,

SoiC (a) une suite horne de ra'els, On pose u miI

L uzl. 2) MAntrer que lu I --h Montrer que un -- O faiblement.

a i e.

;;.27 f

Snit Dcil un sous-eusemble tel que i P espace veatcyd.el engendr

.lait dense dans H.

L.
Stt CE n
) uno

suite de sous-espdces vect0Tieis ferral.s deux a deux ortho-

k5a pese
Oft

uppose que

IiP

131 2 .1u1 2

VuE D..

Moiltrey que 1{ .est recele Hilbertienne des (E n.). >

1-1 07:S i;11 suppgse nue H est se arable.

Vel-t

sglie-ea:ace ,lector gel dense fans U,

' .cicnrret que V reilltD2111 une basa Hilbertiet-Lne de 11. n $oit (en) une s'Ate orthonenne, (e. ,e.) "0 si i
j et je.I al

Montrer 4ue l'on pelle etimplilter les (en) en une base Iiilbertienne.

u
1.2r1 5 Un
Snit
ht7.71:0 de. C2.athendeck

Urt espane :T'Asur avec

<t, Scit 3 p <T ,

50115-espaCC vCCELFriEl fErm. da LP(I) > Cul 5uppose que Eat.. (2). Cla se propone de izontrer que dita E <;1. L) Hntrer qu'II 2xiste une constante C tetle que

IIuI

.Q Chin

VuE1.

[ 011. guarra appliquer la remarqua II.5

2) PrQuver

eHlste U11 .:011.Lazae < cIk II2 Qui te

ilL1L12 qUe

+uee.

Ion pouyra distinguer ies CO5 1 4 ;15 1;2 et 2<p<ca l.

ac
re.A.

3) En d aire que E est un s*us-espace evin ele L2(0.


On " pilo"que dial E

et

r, introduit dafis E une $ulto orthenormdia

Ille.)..pour le produit scalaive de L2(9). On fixe un entilr k, rMonk:Ter qu'i1 exista wiCP I'Lglisee111.a tal que 1: 1 ,12 k r z . CC .it. (X) iq.'[ ( .Z al i si:{G rl \ u, va - Ni ,Iy 2 , .-. ;ale ) ER11 .
i-1 1 1 .1..1 1

[CquioGneer 5521 con.sdIrer ntEigkl,

k
5) En chIduire que
iml

le-(x)1 2 <112 ''9',ten's.w


L

) Concture.

L L

tl

u
-"-1:1F ~va

r ?,
(i-j 1 .1Z.1

I-- e#4,

(1 .LC1

k_

2111 r i

cr e_

'11
%

-J_

s Crt-

:1) 1 2 (1'H

o
Ji

.r-

.CA.,91/4,.

- ti

C.Nr.

f.

) O 1
l e-'4"125.

rgie,A

f.(

1 1, -,3

dir '

1/.,,2_ .......Q.,1 1 -fi.


u.

24r- d. _

-P.< z , j)2.
,
. c

-.._92--

\-- -, )e.4...4..t...5,

---

) [ o -1

")

-T1L--n j -9 O

c4.

11-Q a,5
C

VA:7" -W

e-

b. 1 ;

o, G.76c.)

..,-, . -5 )

s(.70,:ei

el.... .

c.i...,t...

t' :=5

..u...-kr.i..i..Lie_ uz.e...:...

.,( ,1

,. r. . _ t)14

i _ }1...i.

)1z

.. ,
r-4
1_1

te-

Cit

4-19, ,1.-L
Cr <
4

'11

1
ti

cRAPITRE

ZAR Pour montrer que (t,21,r) I 2(u,v) appliquer grPsalo lvec a =


f v el th v,

r identiLl du paralllu-

2) Forrder (i)

+
(Xu1 v)

3) NOter. grZce la dfinition de (,), que l'applicetion t confirme.

Soient A,BcS1 mesurables avec A 1 -1B e'....1-1. " if:er que A et B axis tent) , Soient u ii-xA et v I
rkt nurbpoee d r abord que 1 dcmc 11+v11 + liu-v ti 2 .2(A14.1B1)21P .
011 a Il u iI

et O< IAl

<.=

-r-

P r:
p '

Al 4

!El e t

r.

rau tre par:: 2 (II u: 2 + vil 2 ) P Enfin 011 note que

2( I A i 2/P + 1E11 2/P)

L 1.

01+6) 21p >112/13 +82/p


ca+03 2,5 <112/p +13 2/p

1uJ>0 3i 1 , <2 Vci,B>0 ?) 2.

L
L

On

les n'Aries fonctions ut v pelar ls cas a MI

V.3 1 011 vrifie que (*) Dl ot 2(t u -t,u uto) ti n C dduit


(t - tca) n (t

+t ) lu um12 + (t -t )(1u 12-111 :2) n rn n ni n uh .

(lun 1 2 -1urn 1 2 )<0 Yai,n et 1%1 I%a l i.;er mulo s


ra

) Se ent n>ttt, alurs tn> u =u sr:ice (L)). n

alars

D'aukre part on

pour n>n, n

41 .. 11

I
kJ

(tr rt nun-u i 2 <(t-t )(1u n m m et done

- -5 2 -i4 )44--1---e 1 2 ) (4*t.-

lun.-J.1241 um1 2 - lun1 2

J1 en r6sulle que jun in quand n453. ..1..t (lari) est de Cauchy. 2) Soient n>ri, alors t >tn at lu m 1 l u 1. Pour n>m on a I n m 4t +t ) 1L u -u 1 2 n m n m 0. 0.: donc (t.iri-tr4)(iu i 2--iu j 2 y<(t.--4-e--)-(-1-n-12+---1-2-1. n 71 TI1 n lo n - M- k4,.. I iu -u 2 % n mi On a l'allernative (i) ou bien u , n ou .quand n+= j,
i Lbn I

a - lum 1 2 4

u
fl

lun ift <=, quarid nfw, et dans ce cas (un)est de Catichy et s nutre part, si l'Ion pose si = t n n nun (s. n
Tl

I ,ftri on a

-a y

PI 11

-y )<O, nm

et don. (y) :onverye d'aprs 1a question i), en en dduit que si t -+ t>0 (u s cunverle, n les cas (i) et (i.i) peuvent se produire. er'rea:?.re, par

Cr!:;:n si t Q..xcepT,e,

et (i) un =Citi, (a) un =C.

t.'.oter yie 1.1.-u1 2 =iv-fl 2 - lu-ii 2 .:1, 2(1-u v-u)

o
1 0-1 1

v .s
1) Soit

nK n

; on

montrer

un. --""

f. X
-

La suite d = 1f-u 1 gl dist(f K ) est creissante et eriajorItt. Donc n n dn Appliquant Videntit du paran :Logra:me ({,ver'- a =f-u el: b =f-u.va) on n obtient 2 V +LL-I U nr :9 1:In 1 1 E nn i 2 4- If-tim1 2 ). GJUS:P5%...e". 2 j 27:

on en dduit que lu -u 1 2 <2(d 2-d2 ) si. rri>rk. n nm Done u

kor

et )11 est cisir que uE C. D l autre pare orb a I f-uni lf-vi VvE Ku WEK e.t la limite I f-ul < *C K.

partieulier on a If-urti <

2) Soit Xffi 1717 ; K est convexe (pourqua( 7) n n va contrer que u u=P f 1.e suite d if-un ii.dist(f,KeL ) est dcrossante et done d n -+ t.

D'ilutre part on a ici

-d 2 ) si oi'n in -u 1 2 <2(d2 ri m m eL done u -- u al,rec uE K. n Enfin on note que


g 4 ) i.L

VvEil.n si m>n et J la limite (quand

vient If-u1<tf-v1 siveiptrh . La suite (a n) est dOcroissante et done converse 4C1:: une lmite notge a. noitrons que a winfip. D'abord il est nlair que Inf 1,P < Lo et dona :33eip ez, r t autre part soit u.EK et u:1C 1: ,;nita on ,111.1t.:1..ent ce.<IP(u) (car
ti P K

Alors * <tp(u ) et la ti a .

u). 1:111 a donc tobli que, cc <.Inf

Contidlrons, par exemple, 1e caz On Tu-Tv 11 < car u-v +Ilv11 < !I u-v11
l

Hun

I ot kvil

I.

1177 V 114 +
.

so"

11(.111) 75i - II

11

) 11 ex&Y iN1, tf)

- 1 <21u-911

2) saient u =(1,0) et y-(1,1:1). Atore rru.-Tv Hal et Il u rll =

Cr' chaisit alors o*CI, arbitrairement voisin de O.


3) 'E

coincide avec DE P, . 1.1 suffit de vrifier que si Hui >I, atora (11- ri bl a.1) "s'y E B EI

Wr

.9 "P.e ":"

V. i61

,10

i) AN. (ii)

1er it que r(u.)<' . ((1-thi Ly)

vt

II(,

14: E K,

ce qui implique [Flu On nbtient (ii) en passant La . On remarque que Flv) En efZet, la fonet ion rig ;I ( ) LKO) > W1 (0) F (u) (1" r (u) ,.t-u) 4p(t) F
VIJ , E H.

(v-u)) - F(01 quand t

>0.
O.

t(u-11)) est convext, da eles se 01 e t


r

y. i 2 I

.3 ese sui jettive si at eialeTrient S i E est complet. transpr te R (o) in produit .s.calaire de. ((el { u cdv ) )) (u,v i.t E E. posant

1)

On dot.: que

Dehl r,

proi.priger p.ar ccntinuit e t denzit.1 Le produit scalaire, 1 )) I

deviint ainsi un espace ce ,1k:bgr Y. Z) Spit t E ; 1 appl iva.t jou ,1E 11(0) <f $ .7-1 (1) > dfirlit 1,3^a fume

)inlaire continu sur R(o) que t i cn peut prolengar (pa c ccntinui t1) 411.7) , alerrs :L e thUirl:medeRiIsz-Frchet da ns. R(.71 Gin trouve LL7L 1 127rte nt 11 .1 171 teL .que b !.g)) -dite <15
1-1

u
r T;

.f,,o-1.(1)> VsER(d),
siiivE

((h.d(v))) 1.<

" >

<

facie f eM),, avec, R f,1F) denso. da 7

w-Sir 3) Or a cunstruit une isarr:l:rie a lE


complet. Ceci ignine que

est le enmplIt d E

(a un ilcmcuphismn prh),

1). On va .1tabll.r l ' identit du paralllogymne, Sulent Ee F(0, g E Kv) ; alrs f 1,C - 17 (u v. ) e t done i1 v intit <1+8, u+v > = 1 u+v11 2 < f -g, u-y> = 11 u-y II

1 1

2(1u11 2 + 11v11 2 ) 2) soic a :E --+

11;1+0 2 +

j 14

3:application introduite . l'exercice V. i2 Abre

F(U) {r1(0). En effet 1.1 est olair que a(v)EF(u). [Vali:7re part, on sait que E' un Hilbert pour la norme 9 H E , . En particulter E t est strictement con-

1(u) est rduit a un llment. vexe et done (voir Exercice 1.1) 1

le. 11 [2

ingalit a(tu+ (L-t)v,e11

( 1 -t)

<

ra(upu) + (1 -t)e(v,v) qui-

\mut t(1- t)3(u-v,,u-v) >O. Identinant H et 11' on i ntroduit 1 aplrateur A E (H,11) te 1 que a(u,v)=(Au,v) Alors F`(u) Au +
; en effet on r(u4h) - F(u) (AktA*uh) Vuv11.

a(hh),

r
T:C

v.15

on conmence par prolonger T par c2ntinuf.tE en un oprateur


Jti

F, On rase encuite SET , P 4 F est la projectinn de 5! sur I.

iTi,71T]
(ii) (). Lllyputh1se (t i) entraiue que T est iu3ectiE et que k(T) est
ferr. D i autre para R(T) admet un supplmentalre topololique (puisque H ese un Hilliert). On peut alors appliquer le tlinrIme II,I1ur; COn.c.1 u re quu T admet un inverse a lauche.

(i)

L'Lypothhe

entrame que T est injectif et que R(T) est

form. On en dIdult

gace au thorIme 11.20.

L
Neter que Liuzsuplu n-u1 2 I limsup Clu . 1 2 - 2(un,u) +11.119 <0.

r,

-r

mar

gEn:11~rnn-tmaz~~-nlp-i -o-tpfly ,..,-.~

1".i :20 1) 5t u111(5) on a (Sv,v-u) O Vv H ; templagant v par tv on voit que


(Sv pu)

u
1 ion a (v) ) > O Vv E H ; renplagant v par tv

20 Vvell.
Inversement si uE R.(5)

on volt que (Su, v) O VvE14. ausei le prohibe 141. 2) Appliquer Lax-ililgrata (coraltaire V.8). 3) bithode a). On pose ut (I..t.5)-1 f, Si f E N(S), alors ut = f, Vt >0. Si f R(s) un cr. t f 25v et Von a ut +S(tur-v) I 0 ; I I ' O ; cn dduit que ut pour tout fe1 S) Van dIduit

que,(u t' tu t-v)<0 et iu ti Par consquent

(prkiscr le raisonnerrient) Dans Le ras gnral o f EH, ost dcompose f f ina vec
ti

9111 br'-` 414-1 ' ii tr --i- u faiblement et par suite Su 2 O, c'est--1-dire u e


Drapt,s ta question 1) on a (Su. ,v) t
1

f f n E2 2P - R(5) .2.1 Mthede b). 0n a ut + tSut .. E et dono ki t ' <111. en peut suppbser que

No) .

V-.- 1 1...;(5). D' ea l' on (1.1.duit que


'tO

4-111o v) -o Vvet;(5) et P t.$).f et

la limite (E--..1,1r,I

Vv(" ...4(S). Par conoequent u GrgLiaci. ). 1 13 )`

J
3
SI

die la 'Limito tntrnirte que ut

D'autre par( on a (Sut ,ut) >O, t sup .sw t 1 2 < (E,u) - 114 2

On conclu que ut h u fortement.

(421

1) Poser S = I -T et app-liquer la question I) de l'ex,Ircice V. 2) Ecrire f 2 u - Tu et noter que O (1) n 3) V1riiier d i sbord que lin ern (f) N 0
tVidx. E tant

1n u).
Vf E

donn E EH, att dcompose f 2(1 ta avec fy

N(1-7). et

f2 e Nci-Tri =a(1-T)
On a alors o (f) =13 ,1( 1 1) +cr11 kt2) atl l'un k:11:2; n

4) Appliquer succes3ivement 1' ini'galit (') u, Su, S'u, ... ,s u, ...


r

(aire la some. Noter que

S ISnu- n+l v

iu_ s i+ I u

5) On crit E -u-Tu el 2(,2 -Su) et on obtient hin .6) rroelder mmne la question 3)+

co'

--------

. k1

1-7:131
2) Soit m>. n On applique l'exereiee
4.1

avee f

, v*tP

K u et on

obeient 1P u -P K n 12 riiKu -um 1 : - 1P1..0in -U a n 1PKun-un I 2 - Pxkin-Ilm 12 Done la suite (PKu ) ezt de Caueliy. n 3) Un suppose que u --k u faiblement. On note ve nr,c. Vv E K 1:1-Ln."?Kuripv-PKurb)0
r

ut pags4nt A la lione sd.vhat


r

1-9. 1 v-1) <C il viont (7et d'uctern u

'tvsC. L'unieltd dri

Ceame uq".Z., et peut eFscisir ).n;.te entrohl9 que un 4) Pcur tcut ewi.ste Vv ,v )?.. faiblement pwe r(v)

1 in ft:

1. 1"

et on vett TIC liii (t.t n-11111

NYVEI'd et tor;.t +p(z) (u,z) on eu dduit que 1.p(z) - lira (u ,z) existe nrqg zrare ai. thUrIme de Rlfloz-l-rchet. EriFin cu note que (u-a,v-t) 0 VveK et done L r P . K 5) j\prls bransi,ithu et 31.1.atation on peut toujouts supposer que K *Bit . Dono lu i 4 a . n

.I L

Si a< S i u.>

hig.r.4 u or. trote

partir d'un eertain rata ;. ?u n lu Y.. a 't converge fortement et VvE.g. et done (u -t,v-t.) ge. iti rd atissi.(ur), VvK avec

) Qn 11 (I,T -F u n ti
a --> O (et s

Eipend de v). V'er K avec en ' Cl.

par zdditiein. nr oJC1,211 Ill a-tsv-1:21 <en


u

U
1~1,i1111").
L

-"r-

" "r

^~73111M.

Si eF
Di

--4. 0 faiblertent, licirs d' E X. et On a (o -1,v- Z) <O de La limite eDtratrie que .,,n. -'.- t faiblement.

V-JE K. u' rya

.i. ,..,,...,
1-7 1. 1T

Natr qu

n 'u

est borni et peJur cheque j rIxe., twilu ,.)> O. n

[V ..27. L

Soit F la .rmture de l'espatt vectoriet engendra pt les = (0). Vul


EL

fa1/41: razintrQt. que P =El iu eneute On ,VeD. 1 3


u

qte 9 E 11

1 P RU! 2 =

11 5rui 2

et done P . 5,u l

=P 1

ul

Nfli I) ;

Fi

5771

1) Ort 52it

sait que (y

est sparable mit preposition 111.22). de V dammbrabIe t demsa. C,c ptt.eade encuite

) un

54:11j -elnietine

la dImonstration. du thIorItri.e V.10,


-

7771

7-) Si 2 <p ocer qu s(17let


-D-11 t.924..t.

[in L.itiline 1 ittlgalitl.

1-2i13 ..t

Z1u1 2ill

'

espata d 11.ilbett (EiC!pard.ble 11.1.1 tren) da dier.orisi..Dn.

suite

grthannrml .e, aktt Migt

5) En intgpant 5uw 0 nn nbrient de diga E.

fl

iI

ti

1 1ir

_
o

--

r i
F.ZZ, eillil....L.

h,

1 II 1,11,.

wr if

., . .

1_,C t.-.,,.2

(./ .) ea

L. le

'Ir:'....S 11.7=

'IN" ) +? ' "

r..k.... II- 4.-

1I1 ,

. . . 't e- -

UD i

t.'"

e 1-k r ....

tz .. -..

- .-=

2,5.1.::::. ....,,_._

ve g.42-,-,...:. ,_ ....10

...............1, 11..hoz 2....

11 " ) -1 .1 CIL L

rsa._

-.P?_,5 (,....z.,..

-,.,
' -.;:'.-, ......7, 4 .. ... ,,,
r,41,

1 (3 .0.......3, ii.L._ .c. .. F ' ... '--

_,

2 .

(7.:
'V ..,...-Q. ......."%

.L5..2' %

4.

...

4-

G 1.

r'l - CI

- --- ., _.. . v.1-1- LGP

11,,ok."--1- ,

L
^ruTr-ffinwr-a --11;ipLg7rw.-,r

C, 1 %al

kez
L(2%.

kr

oj.

0 r

zfj.

"`"

.c1,1

ct_

25.

s.

de.s,

e2.5
221 "

LP

"

'E -71

c-

ci4:14.1

L.0

" h

rI

"S

lir

,
L

Ft

e_?:'

e_
ir

r _ .........
1) .' C : - '..

.0 7........1, (..?..1...

....%t-_,.........r c-Lt..... le....

-ird._

(,A.'ri, ) lo, ... "


_ _.

rr i

-.1" 5,-.1

Z:1 -.0:::i."6., 0._ 6. . <I -

1...-....k. t...

,J-21,_

..,p-1;__. C (1

(1., ,,,I ) ,, a.o


CoL

7 %-%-'

%-k.ArL L.

-e-cfr,4. e.... . _ i-C 717,.. I , .i .., _ _ ....

Lk:kr, . ,L.,

,: -.'" N ''' S- .0 1.1 0 11 1 %

.r. _ 5, ..1.1.....1.4. Q .1.....'

,--e-.,_ (.....: cz, b-- kr

r
r-

Vous aimerez peut-être aussi